You are on page 1of 124

 

KAMKUS COLLEGE OF LAW


B.A.LL.B. VIIITH SEM
INTELLECTUAL PROPERTY LAW
CODE BL-(803)
UNIT-1st

LONG QUESTION:
Q.1 .what is intellectual property and what are the kinds of intellectual
property?

Ans-Intellectual property refers to creations of the mind: inventions; literary and


artistic works; and symbols, names and images used in commerce. Intellectual
property is divided into two categories:

• Industrial Property includes patents for inventions, trademarks, industrial


designs and geographical indications.
• Copyright covers literary works (such as novels, poems and plays), films,
music, artistic works (e.g., drawings, paintings, photographs and sculptures)
and architectural design. Rights related to copyright include those of
performing artists in their performances, producers of phonograms in their
recordings, and broadcasters in their radio and television programs.
Intellectual property rights-

Intellectual property rights are like any other property right. They allow creators,
or owners, of patents, trademarks or copyrighted works to benefit from their own
work or investment in a creation. These rights are outlined in Article 27 of the
Universal Declaration of Human Rights, which provides for the right to benefit
from the protection of moral and material interests resulting from authorship of
scientific, literary or artistic productions. The importance of intellectual property
was first recognized in the Paris Convention for the Protection of Industrial
Property (1883) and the Berne Convention for the Protection of Literary and
Artistic Works (1886). Both treaties are administered by the World Intellectual
Property Organization (WIPO).

 
 

Why promote and protect intellectual property-

There are several compelling reasons. First, the progress and well-being of
humanity rest on its capacity to create and invent new works in the areas of
technology and culture. Second, the legal protection of new creations encourages
the commitment of additional resources for further innovation. Third, the
promotion and protection of intellectual property spurs economic growth, creates
new jobs and industries, and enhances the quality and enjoyment of life.

How does the average person benefit-

Intellectual property rights reward creativity and human endeavor, which fuel the
progress of humankind. Some examples:

• The multibillion dollar film, recording, publishing and software industries –


which bring pleasure to millions of people worldwide – would not exist
without copyright protection.

• Without the rewards provided by the patent system, researchers and


inventors would have little incentive to continue producing better and more
efficient products for consumers.

• Consumers would have no means to confidently buy products or services


without reliable, international trademark protection and enforcement
mechanisms to discourage counterfeiting and piracy.

Kinds of intellectual property right-

• patent
• trademark
• copyright
• industrial design
• geographical indication

 
 

• Patent-
A patent is an exclusive right granted for an invention – a product or process
that provides a new way of doing something, or that offers a new technical
solution to a problem. A patent provides patent owners with protection for
their inventions. Protection is granted for a limited period, generally 20
years.
Why are patents necessary- Patents provide incentives to individuals by
recognizing their creativity and offering the possibility of material reward
for their marketable inventions. These incentives encourage innovation,
which in turn enhances the quality of human life.
• Copyright-
Copyright laws grant authors, artists and other creators protection for their
literary and artistic creations, generally referred to as “works”. The
beneficiaries of related rights are: performers (such as actors and musicians)
in their performances; producers of phonograms (for example, compact
discs) in their sound recordings; and broadcasting organizations in their
radio and television programs. Works covered by copyright include, but are
not limited to: novels, poems, plays, reference works, newspapers,
advertisements, computer programs, databases, films, musical compositions,
choreography, paintings, drawings, photographs, sculpture, architecture,
maps and technical drawings.

• Trademark-
Trademark defined under Section 2 (zb) of the Trade Marks Act, 1999 as,
"trade mark means a mark capable of being represented graphically and
which is capable of distinguishing the goods or services of one person from
those of others and may include shape of goods, their packaging and
combination of colors." A mark can include a device, brand, heading, label,
ticket, name, signature, word, letter, and numeral, shape of goods, packaging
or combination of colors or any such combinations. trademark protection
ensures that the owners of marks have the exclusive right to use them to
identify goods or services, or to authorize others to use them in return for
payment.

 
 

• Industrial design-
An industrial design right is an intellectual property right that protects the
visual design of objects that are not purely utilitarian. An industrial
design consists of the creation of a shape, configuration or composition of
pattern or color, or combination of pattern and color in three-dimensional
form containing aesthetic value. An industrial design can be a two- or
three-dimensional pattern used to produce a product, industrial commodity
or handicraft. Industrial design laws in some countries grant – without
registration – time- and scope limited protection to so-called “unregistered
industrial designs”.Depending on the particular national law and the kind of
design, industrial designs may also be protected as works of art under
copyright law.

• Geographical indication-
A geographical indication is a sign used on goods that have a specific
geographical origin and possess qualities or a reputation due to that place of
origin. Most commonly, a geographical indication consists of the name of
the place of origin of the goods. Agricultural products typically have
qualities that derive from their place of production and are influenced by
specific local geographical factors, such as climate and soil.geographical
indications may be used for a wide variety of agricultural products, such as,
for example, “Tuscany” for olive oil produced in a specific area of Italy, or
“Roquefort” for cheese produced in that region of France. The use of
geographical indications is not limited to agricultural products. They may
also highlight specific qualities of a product that are due to human factors
found in the product’s place of origin, such as specific manufacturing skills.

Q.2 Define intellectual property and explain its nature?

 
 

History of IPR

IPR is not a new concept. It is believed that IPR initially started in North Italy
during the Renaissance era. In 1474, Venice issued a law regulating patents
protection that granted an exclusive right for the owner. The copyright dates back
to 1440 A.D. when Johannes Gutenberg invented the printing press with
replaceable/moveable wooden or metal letters. Late in the 19th century, a number
of countries felt the necessity of laying down laws regulating IPR. Globally, two
conventions constituting the basis for IPR system worldwide had been signed;
Paris Convention for the Protection of Industrial Property (1883) Berne
Convention for the Protection of Literary and Artistic Works (1886).

Definition:

Intellectual property is the product of the human intellect including creativity


concepts, inventions, industrial models, trademarks, songs, literature, symbols,
names, brands,....etc. Intellectual Property Rights do not differ from other property
rights. They allow their owner to completely benefit from his/her product which
was initially an idea that developed and crystallized. They also entitle him/her to
prevent others from using, dealing or tampering with his/her product without prior
permission from him/her. He/she can in fact legally sue them and force them to
stop and compensate for any damages.

Protection of Intellectual Property Rights

Protection of IPR allows the innovator, brand owner, patent holder and copyright
holder to benefit from his/her work, labor and investment, which does not mean
monopoly of the intellect. Such rights are set out in the International Declaration of
Human Rights, which provides for the right to benefit from the protection of the
moral and physical interests resulting from the right holder’s work; literal or
artistic product.

 
 

-Intellectual property refers to creations of the mind: inventions; literary and


artistic works; and symbols, names and images used in commerce. Intellectual
property is divided into two categories:

• Industrial Property includes patents for inventions, trademarks, industrial


designs and geographical indications.
• Copyright covers literary works (such as novels, poems and plays), films,
music, artistic works (e.g., drawings, paintings, photographs and sculptures)
and architectural design. Rights related to copyright include those of
performing artists in their performances, producers of phonograms in their
recordings, and broadcasters in their radio and television programs.

Intellectual property rights-

Intellectual property rights are like any other property right. They allow creators,
or owners, of patents, trademarks or copyrighted works to benefit from their own
work or investment in a creation. These rights are outlined in Article 27 of the
Universal Declaration of Human Rights, which provides for the right to benefit
from the protection of moral and material interests resulting from authorship of
scientific, literary or artistic productions. The importance of intellectual property
was first recognized in the Paris Convention for the Protection of Industrial
Property (1883) and the Berne Convention for the Protection of Literary and
Artistic Works (1886). Both treaties are administered by the World Intellectual
Property Organization (WIPO).

Why promote and protect intellectual property-

There are several compelling reasons. First, the progress and well-being of
humanity rest on its capacity to create and invent new works in the areas of
technology and culture. Second, the legal protection of new creations encourages
the commitment of additional resources for further innovation. Third, the
promotion and protection of intellectual property spurs economic growth, creates
new jobs and industries, and enhances the quality andenjoyment of life.

 
 

Intellectual property rights reward creativity and human endeavor, which fuel the
progress of humankind. Some examples:

• The multibillion dollar film, recording, publishing and software industries –


which bring pleasure to millions of people worldwide – would not exist
without copyright protection.

• Without the rewards provided by the patent system, researchers and


inventors would have little incentive to continue producing better and more
efficient products for consumers.

• Consumers would have no means to confidently buy products or services


without reliable, international trademark protection and enforcement
mechanisms to discourage counterfeiting and piracy. Nature:

• Intellectual property rights include copyright, patent, trademark, geographic


indication of origin, industrial design, trade secrets, database protection
laws, publicity rights laws, laws for the protection of plant varieties, laws for
the protection of semi-conductor chips (which store information for later
retrieval) ...

• Among the bundles of intellectual property rights, copyright that deals with
the protection of literary, artistic and scientific works is one. Intellectual
property, very broadly, means the legal property which results from
intellectual activity in the industrial, scientific and artistic fields.

Why is Intellectual Property Important

Intellectual property protection is critical to fostering innovation. Without


protection of ideas, businesses and individuals would not reap the full benefits of
their inventions and would focus less on research and development.

Nature:

 
 

Intellectual property rights include copyright, patent, trademark, geographic


indication of origin, industrial design, trade secrets, database protection laws,
publicity rights laws, laws for the protection of plant varieties, laws for the
protection of semi-conductor chips (which store information for later retrieval) ...

Among the bundles of intellectual property rights, copyright that deals with the
protection of literary, artistic and scientific works is one. Intellectual property, very
broadly, means the legal property which results from intellectual activity in the
industrial, scientific and artistic fields.

Q.3. define the concept of intellectual property explaining various intellectual


rights available to the people?

Ans- Intellectual property refers to creations of the mind: inventions; literary and
artistic works; and symbols, names and images used in commerce. Intellectual
property is divided into two categories:

• Industrial Property includes patents for inventions, trademarks, industrial


designs and geographical indications.
• Copyright covers literary works (such as novels, poems and plays), films,
music, artistic works (e.g., drawings, paintings, photographs and sculptures)
and architectural design. Rights related to copyright include those of
performing artists in their performances, producers of phonograms in their
recordings, and broadcasters in their radio and television programs.

Intellectual property rights-

Intellectual property rights are like any other property right. They allow creators,
or owners, of patents, trademarks or copyrighted works to benefit from their own
work or investment in a creation. These rights are outlined in Article 27 of the
Universal Declaration of Human Rights, which provides for the right to benefit

 
 

from the protection of moral and material interests resulting from authorship of
scientific, literary or artistic productions.

The importance of intellectual property was first recognized in the Paris


Convention for the Protection of Industrial Property (1883) and the Berne
Convention for the Protection of Literary and Artistic Works (1886). Both treaties
are administered by the World Intellectual Property Organization (WIPO).

Why promote and protect intellectual property-

There are several compelling reasons. First, the progress and well-being of
humanity rest on its capacity to create and invent new works in the areas of
technology and culture. Second, the legal protection of new creations encourages
the commitment of additional resources for further innovation. Third, the
promotion and protection of intellectual property spurs economic growth, creates
new jobs and industries, and enhances the quality and enjoyment of life.

Intellectual property rights reward creativity and human endeavor, which fuel the
progress of humankind. Some examples:

• The multibillion dollar film, recording, publishing and software industries –


which bring pleasure to millions of people worldwide – would not exist
without copyright protection.
• Without the rewards provided by the patent system, researchers and
inventors would have little incentive to continue producing better and more
efficient products for consumers.
• Consumers would have no means to confidently buy products or services
without reliable, international trademark protection and enforcement
mechanisms to discourage counterfeiting and piracy.
Kinds of intellectual property right-

• patent

• trademark

• copyright

 
 

• industrial design

• geographical indication

• Patent-

A patent is an exclusive right granted for an invention – a product or process that


provides a new way of doing something, or that offers a new technical solution to a
problem. A patent provides patent owners with protection for their inventions.
Protection is granted for a limited period, generally 20 years. Why are patents
necessary- Patents provide incentives to individuals by recognizing their creativity
and offering the possibility of material reward for their marketable inventions.
These incentives encourage innovation, which in turn enhances the quality of
human life.

• Copyright-

Copyright laws grant authors, artists and other creators protection for their literary
and artistic creations, generally referred to as “works”. The beneficiaries of related
rights are: performers (such as actors and musicians) in their performances;
producers of phonograms (for example, compact discs) in their sound recordings;
and broadcasting organizations in their radio and television programs. Works
covered by copyright include, but are not limited to: novels, poems, plays,
reference works, newspapers, advertisements, computer programs, databases,
films, musical compositions, choreography, paintings, drawings, photographs,
sculpture, architecture, maps and technical drawings.

• Trademark-

Trademark defined under Section 2 (zb) of the Trade Marks Act, 1999 as, "trade
mark means a mark capable of being represented graphically and which is capable
of distinguishing the goods or services of one person from those of others and may
include shape of goods, their packaging and combination of colors." A mark can
include a device, brand, heading, label, ticket, name, signature, word, letter, and
10 

 
 

numeral, shape of goods, packaging or combination of colors or any such


combinations. Trademark protection ensures that the owners of marks have the
exclusive right to use them to identify goods or services, or to authorize others to
use them in return for payment.

• Industrial design-

An industrial design right is an intellectual property right that protects the visual
design of objects that are not purely utilitarian. An industrial design consists of the
creation of a shape, configuration or composition of pattern or colour, or
combination of pattern and colour in three-dimensional form containing aesthetic
value. An industrial design can be a two- or three-dimensional pattern used to
produce a product, industrial commodity or handicraft. Industrial design laws in
some countries grant – without registration – time- and scope limited protection to
so-called “unregistered industrial designs”. Depending on the particular national
law and the kind of design, industrial designs may also be protected as works of art
under copyright law.

• Geographical indication-

A geographical indication is a sign used on goods that have a specific geographical


origin and possess qualities or a reputation due to that place of origin. Most
commonly, a geographical indication consists of the name of the place of origin of
the goods. Agricultural products typically have qualities that derive from their
place of production and are influenced by specific local geographical factors, such
as climate and soil. geographical indications may be used for a wide variety of
agricultural products, such as, for example, “Tuscany” for olive oil produced in a
specific area of Italy, or “Roquefort” for cheese produced in that region of France.
The use of geographical indications is not limited to agricultural products. They
may also highlight specific qualities of a product that are due to human factors
found in the product’s place of origin, such as specific manufacturing skills.

11 

 
 

SHORT QUESTION:

Q.4. Explain patent and its kinds?

Ans-Patent defines under Indian patent act-sec 2(m)-“means a patent for any
invention granted under this act”. A patent is an exclusive right granted for an
invention – a product or process that provides a new way of doing something, or
that offers a new technical solution to a problem. A patent provides patent owners
with protection for their inventions. Protection is granted for a limited period,
generally 20 years. it maintain by paying the renewal fees at every succeeding
year. A patent is not granted for an idea or principle as such, but it is granted for
some article or the process of making the same article.

A patent example is a sample of a patent that has already been granted to someone.
A patent example is useful for someone who is applying for a patent and wants to
know what information is necessary to include in their patent application. A patent
is granted by the U.S. Patent and Trademark Office or USPTO.

Kinds:

There are three kinds of patents:

1.) utility patents

2.) design patents

3.) plant patents

Each type of patent has its own eligibility requirements and protects a specific
type of invention or discovery; however, it's possible for one invention or
discovery to potentially have more than one type of patent available for it.

Q.5. Explain Geographical indication?

A geographical indication is a sign used on goods that have a specific


geographical origin and possess qualities or a reputation due to that place of
12 

 
 

origin. Most commonly, a geographical indication consists of the name of


the place of origin of the goods. Agricultural products typically have
qualities that derive from their place of production and are influenced by
specific local geographical factors, such as climate and soil. Geographical
indications may be used for a wide variety of agricultural products, such as,
for example, “Tuscany” for olive oil produced in a specific area of Italy, or
“Roquefort” for cheese produced in that region of France. The use of
geographical indications is not limited to agricultural products. They may
also highlight specific qualities of a product that are due to human factors
found in the product’s place of origin, such as specific manufacturing skills.

Q.6 Define design and plant design?


Ans-An industrial design right is an intellectual property right that protects
the visual design of objects that are not purely utilitarian. An industrial
design consists of the creation of a shape, configuration or composition of
pattern or color, or combination of pattern and color in three-dimensional
form containing aesthetic value. An industrial design can be a two- or
three-dimensional pattern used to produce a product, industrial commodity
or handicraft.
Industrial design laws in some countries grant – without registration – time-
and scope limited protection to so-called “unregistered industrial designs”.
Depending on the particular national law and the kind of design, industrial
designs may also be protected as works of art under copyright law.

Plant design:
Plant variety protection, also called a "plant breeder's right" (PBR), is a form
of intellectual property right granted to the breeder of a new plant variety.
According to this right, certain facts concerning the exploitation of the
protected variety require the prior authorization of the breeder. Plant variety
protection is an independent sui generis form of protection, tailored to
protect new plant varieties and has certain features in common with other
intellectual property rights.
13 

 
 

VERY SHORT QUESTION:

Q.7. what is artistic work?

Ans- Artistic work” means a painting, a sculpture, a drawing (including a


diagram, map, chart or plan), an engraving or a photograph, whether or not any
such work possesses artistic quality; a [work of architecture]; and any other
work of artistic craftsmanship; as provided under Section 2(c) of the Copyright
Act.

Donoghue v Allied Newspaper Ltd [1938] Ch. 106 have interpreted an


Author as the person who created the work by putting it into a form that is
protected by copyright. Exceptions to this include:

• Employees – if the work is created as part of their employment, then the


employer will usually own copyright;

• Commissioned works – it is possible for a person to own copyright for


works created by someone else where the creation of those works are
commissioned by them;

• And governmental works – if works are created for the Government, the
Government will generally be the copyright owner.

Q.8. what do you mean by trade mark?

Ans-Trademark defined under Section 2 (Zb) of the Trade Marks Act, 1999 as,
"trade mark means a mark capable of being represented graphically and which
is capable of distinguishing the goods or services of one person from those of
others and may include shape of goods, their packaging and combination of
coolers." A mark can include a device, brand, heading, label, ticket, name,
signature, word, letter, and numeral, shape of goods, packaging or combination

14 

 
 

of colors or any such combinations. Trademark protection ensures that the


owners of marks have the exclusive right to use them to

identify goods or services, or to authorize others to use them in return for


payment.

Q.9. What are the object of trademark?

Objective:

1) To work out whether using a trade mark will be infringing anyone else’s
right.

2) To work out whether would be able to register your trademark if you were to
make an application.

3) Development in trading and commercial practice.

4) Increasing globalization of trade and industry.

5) To give effect to important judicial decision.

6) The need to encourage investment flows and transfers of technology.

15 

 
 

UNIT- IIND
LONG QUESTION:

Q.1. Explain Paris convention?

The Paris Convention applies to industrial property in the widest sense, including
patents, trademarks, industrial designs, utility models (a kind of "small-scale
patent" provided for by the laws of some countries), service marks, trade names
(designations under which an industrial or commercial activity is carried out),
geographical indications (indications of source and appellations of origin) and the
repression of unfair competition.

The substantive provisions of the Convention fall into three main categories:
national treatment, right of priority, common rules.

(1) Under the provisions on national treatment, the Convention provides that, as
regards the protection of industrial property, each Contracting State must grant the
same protection to nationals of other Contracting States that it grants to its own
nationals. Nationals of non-Contracting States are also entitled to national
treatment under the Convention if they are domiciled or have a real and effective
industrial or commercial establishment in a Contracting State.

(2) The Convention provides for the right of priority in the case of patents (and
utility models where they exist), marks and industrial designs. This right means
that, on the basis of a regular first application filed in one of the Contracting States,
the applicant may, within a certain period of time (12 months for patents and utility
models; 6 months for industrial designs and marks), apply for protection in any of
the other Contracting States. These subsequent applications will be regarded as if
they had been filed on the same day as the first application. In other words, they
will have priority (hence the expression "right of priority") over applications filed
by others during the said period of time for the same invention, utility model, mark
or industrial design. Moreover, these subsequent applications, being based on the
first application, will not be affected by any event that takes place in the interval,
16 

 
 

such as the publication of an invention or the sale of articles bearing a mark or


incorporating an industrial design. One of the great practical advantages of this
provision is that applicants seeking protection in several countries are not required
to present all of their applications at the same time but have 6 or 12 months to
decide in which countries they wish to seek protection, and to organize with due
care the steps necessary for securing protection.

(3) The Convention lays down a few common rules that all Contracting States must
follow. The most important are:

(a) Patents. Patents granted in different Contracting States for the same invention
are independent of each other: the granting of a patent in one Contracting State
does not oblige other Contracting States to grant a patent; a patent cannot be
refused, annulled or terminated in any Contracting State on the ground that it has
been refused or annulled or has terminated in any other Contracting State.

The inventor has the right to be named as such in the patent.

The grant of a patent may not be refused, and a patent may not be invalidated, on
the ground that the sale of the patented product, or of a product obtained by means
of the patented process, is subject to restrictions or limitations resulting from the
domestic law.

Each Contracting State that takes legislative measures providing for the grant of
compulsory licenses to prevent the abuses which might result from the exclusive
rights conferred by a patent may do so only under certain conditions. A
compulsory license (a license not granted by the owner of the patent but by a
public authority of the State concerned), based on failure to work or insufficient
working of the patented invention, may only be granted pursuant to a request filed
after three years from the grant of the patent or four years from the filing date of
the patent application, and it must be refused if the patentee gives legitimate
reasons to justify this inaction. Furthermore, forfeiture of a patent may not be
provided for, except in cases where the grant of a compulsory license would not
have been sufficient to prevent the abuse. In the latter case, proceedings for

17 

 
 

forfeiture of a patent may be instituted, but only after the expiration of two years
from the grant of the first compulsory license.

(b) Marks. The Paris Convention does not regulate the conditions for the filing and
registration of marks which are determined in each Contracting State by domestic
law. Consequently, no application for the registration of a mark filed by a national
of a Contracting State may be refused, nor may a registration be invalidated, on the
ground that filing, registration or renewal has not been effected in the country of
origin. The registration of a mark obtained in one Contracting State is independent
of its possible registration in any other country, including the country of origin;
consequently, the lapse or annulment of the registration of a mark in one
Contracting State will not affect the validity of the registration in other Contracting
States.

Where a mark has been duly registered in the country of origin, it must, on request,
be accepted for filing and protected in its original form in the other Contracting
States. Nevertheless, registration may be refused in well- defined cases, such as
where the mark would infringe the acquired rights of third parties; where it is
devoid of distinctive character; where it is contrary to morality or public order; or
where it is of such a nature as to be liable to deceive the public.

If, in any Contracting State, the use of a registered mark is compulsory, the
registration cannot be canceled for non- use until after a reasonable period, and
then only if the owner cannot justify this inaction.

Each Contracting State must refuse registration and prohibit the use of marks that
constitute a reproduction, imitation or translation, liable to create confusion, of a
mark used for identical and similar goods and considered by the competent
authority of that State to be well known in that State and to already belong to a
person entitled to the benefits of the Convention.

Each Contracting State must likewise refuse registration and prohibit the use of
marks that consist of or contain, without authorization, armorial bearings, State
emblems and official signs and hallmarks of Contracting States, provided they
have been communicated through the International Bureau of WIPO. The same
18 

 
 

provisions apply to armorial bearings, flags, other emblems, abbreviations and


names of certain intergovernmental organizations.

Collective marks must be granted protection.

(c) Industrial Designs. Industrial designs must be protected in each Contracting


State, and protection may not be forfeited on the ground that articles incorporating
the design are not manufactured in that State.

(d) Trade Names. Protection must be granted to trade names in each Contracting
State without there being an obligation to file or register the names.

(e) Indications of Source. Measures must be taken by each Contracting

State against direct or indirect use of a False indication of the source of goods or
the identity of their producer, manufacturer or trader.

(f) Unfair competition. Each Contracting State must provide for effective
protection against unfair competition.

The Paris Union, established by the Convention, has an Assembly and an


Executive Committee. Every State that is a member of the Union and has adhered
to at least the administrative and final provisions of the Stockholm Act (1967) is a
member of the Assembly. The members of the Executive Committee are elected
from among the members of the Union, except for Switzerland, which is a member
ex officio. The establishment of the biennial program and budget of the WIPO
Secretariat – as far as the Paris Union is concerned – is the task of its Assembly.

The Paris Convention, concluded in 1883, was revised at Brussels in 1900, at


Washington in 1911, at The Hague in 1925, at London in 1934, at Lisbon in 1958
and at Stockholm in 1967, and was amended in 1979.

Q.2. Write a note on Berne convention?

19 

 
 

Ans-The Berne Convention deals with the protection of works and the rights of
their authors. It is based on three basic principles and contains a series of
provisions determining the minimum protection to be granted, as well as special
provisions available to developing countries that want to make use of them.

(1) The three basic principles are the following:

a.)Works originating in one of the Contracting States (that is, works the author of
which is a national of such a State or works first published in such a State) must be
given the same protection in each of the other Contracting States as the latter
grants to the works of its own nationals (principle of "national treatment").

b) protection must not be conditional upon compliance with any formality


(principle of "automatic “protection).

c) protection is independent of the existence of protection in the country of origin


of the work (principle of "independence" of protection). If, however, a Contracting
State provides for a longer term of protection than the minimum prescribed by the
Convention and the work ceases to be protected in the country of origin, protection
may be denied once protection in the country of origin ceases.

(2) The minimum standards of protection relate to the works and rights to be
protected, and to the duration of protection:

As to works, protection must include "every production in the literary, scientific


and artistic domain, whatever the mode or form of its expression" (Article 2(1) of
the Convention).

Subject to certain allowed reservations, limitations or exceptions, the following are


among the rights that must be recognized as exclusive rights of authorization:

•the right to translate,

•the right to make adaptations and arrangements of the work,

•the right to perform in public dramatic, dramatic-musical and musical works,

•the right to recite literary works in public,


20 

 
 

•the right to communicate to the public the performance of such works,

•the right to broadcast (with the possibility that a Contracting State may provide for
a mere right to equitable remuneration instead of a right of authorization)

•the right to make reproductions in any manner or form (with the possibility that a
Contracting State may permit, in certain special cases, reproduction without
authorization, provided that the reproduction does not conflict with the normal
exploitation of the work and does not unreasonably prejudice the legitimate
interests of the author; and the possibility that a Contracting State may provide, in
the case of sound recordings of musical works, for a right to equitable
remuneration)

•the right to use the work as a basis for an audiovisual work, and the right to
reproduce, distribute, perform in public or communicate to the public that
audiovisual work.

The Convention also provides for "moral rights", that is, the right to claim
authorship of the work and the right to object to any mutilation, deformation or
other modification of, or other derogatory action in relation to, the work that would
be prejudicial to the author's honor or reputation.

(c) As to the duration of protection, the general rule is that protection must be
granted until the expiration of the 50th year after the author's death. There are,
however, exceptions to this general rule. In the case of anonymous or
pseudonymous works, the term of protection expires 50 years after the work has
been lawfully

Made available to the public, except if the pseudonym leaves no doubt as to the
author's identity or if the author discloses his or her identity during that period; in
the latter case, the general rule applies. In the case of audiovisual
(cinematographic) works, the minimum term of protection is 50 years after the
making available of the work to the public ("release") or – failing such an event –
from the creation of the work. In the case of works of applied art and photographic
works, the minimum term is 25 years from the creation of the work.

21 

 
 

(3) The Berne Convention allows certain limitations and exceptions on economic
rights, that is, cases in which protected works may be used without the
authorization of the owner of the copyright, and without payment of compensation.

(4) The Appendix to the Paris Act of the Convention also permits developing
countries to implement non-voluntary licenses for translation and reproduction of
works in certain cases, in connection with educational activities. In these cases, the
described use is allowed without the authorization of the right holder, subject to the
payment of remuneration to be fixed by the law.

The Berne Union has an Assembly and an Executive Committee. Every country
that is a member of the Union and has adhered to at least the administrative and
final provisions of the Stockholm Act is a member of the Assembly. The members
of the Executive Committee are elected from among the members of the Union,
except for Switzerland, which is a member ex officio.

The establishment of the biennial program and budget of the WIPO Secretariat – as
far as the Berne Union is concerned – is the task of its Assembly.

The Berne Convention, concluded in 1886, was revised at Paris in 1896 and at
Berlin in 1908, completed at Berne in 1914, revised at Rome in 1928, at Brussels
in 1948, at Stockholm in 1967 and at Paris in 1971, and was amended in 1979.

The Convention is open to all States. Instruments of ratification or accession must


be deposited with the Director General of WIPO.

[1] Under the Agreement on Trade-Related Aspects of Intellectual Property Rights


(TRIPS Agreement), the principles of national treatment, automatic protection and
independence of protection also bind those World Trade Organization (WTO)
Members not party to the Berne Convention. In addition, the TRIPS Agreement
imposes an obligation of "most-favored-nation treatment", under which advantages
accorded by a WTO Member to the nationals of any other country must also be
accorded to the nationals of all WTO Members. It is to be noted that does not apply
to national treatment and most-favored obligations.

22 

 
 

[2] Under the TRIPS Agreement, an exclusive right of rental must be recognized in
respect of computer programs and, under certain conditions, audiovisual works.

[3] Under the TRIPS Agreement, any term of protection that is calculated on a
basis other than the life of a natural person must be at least 50 years from the first
authorized publication of the work, or – failing such an event – 50 years from the
making of the work. However, this rule does not apply to photographic works, or
to works of applied art.

[4] It is to be noted that WTO Members, even those not party to the Berne
Convention, must comply with the substantive law provisions of the Berne
Convention, except that WTO Members not party to the Convention are not bound
by the moral rights provisions of the Convention.

Q. 3. Write a note on Trips?

Ans- Is an international legal agreement between all the member nations of the
World Trade Organization (WTO).Came into effect on 1 January 1995. It sets
down minimum standards for the regulation by national governments of many
forms of intellectual property (IP) as applied to nationals of other WTO member
nations.

The Agreement on Trade-Related Aspects of Intellectual Property Rights (TRIPS)


is an international legal agreement between all the member nations of the World
Trade Organization (WTO). It sets down minimum standards for the regulation by
national governments of many forms of intellectual property (IP) as applied to
nationals of other WTO member nations. A TRIP was negotiated at the end of the
Uruguay Round of the General agreement on tariffs and trade between 1989 and
1990 and is administered by the WTO.

23 

 
 

The TRIPS agreement introduced intellectual property law into the multilateral
trading system for the first time and remains the most comprehensive multilateral
agreement on intellectual property to date. In 2001, developing countries,
concerned that developed countries were insisting on an overly narrow reading of
TRIPS, initiated a round of talks that resulted in the Doha members

Agreement on Trade-Related Aspects of Intellectual Property Rights at Wikisource


Declaration. The Doha declaration is a WTO statement that clarifies the scope of
TRIPS, stating for example that TRIPS can and should be interpreted in light of the
goal "to promote access to medicines for all."

Need and Justification of TRIPs

The introductory of TRIPs Agreement clarify the objectives of TRIPs. They may
be summarized as under:

1.To reduce distortions and impediments to international trade by taking into


account the need to promote effective and adequate protection of IPRs, and to
ensure that measures and procedures See, to enforce IPRs do not themselves
become barriers to legitimate trade.

2.) To provide a multilateral frame work of principles, rules and disciplines dealing
with international trade in counterfeit goods

3.) To cater to the special needs of the least developed countries in respect of
maximum flexibility in the domestic implementation of laws and regulations in
order to enable them to create a sound and viable technological base.

4.) To resolve disputes on trade-related intellectual property issues through


multilateral procedures.

5.) To establish a mutually supportive relationship between the WTO and the
WIPO as well as other relevant organization.

6.) To provide for adequate standards and principles concerning the availability,
scope and use of trade related Intellectual Property Rights.

24 

 
 

7.) To provide effective and expeditious procedure for the multilateral prevention
and settlement of disputes between governments.

8.) To provide effective and appropriate means for the enforcement of trade related
intellectual property right, taking into consideration differences in national legal
system.

Outer Framework of TRIPs

Divided into 7 parts containing 73 Articles the scheme of TRIPs can be seen at a
glance as under:-

Part I. General Provisions and Basic Principles (Arts. 1-8)

Part II. Standard concerning the availability, scope and use of IPRs minimum
standards provide to each member country (Arts. 9-40).

Part III. Enforcement of intellectual property rights (Arts. 41-61).

Part IV. Acquisition and maintenance of IPRs (Art 62).

Part V. Dispute Prevention and settlement (Arts. 63-65).

Part VI. Transitional Arrangement (Arts. 66-67).

Part VII. Institutional Arrangements (Arts. 68-73).

Categories of Intellectual Property covered by TRIPs Agreement

The Agreement specifies that the term "intellectual property" refers to all the
categories of intellectual property which are covered under Sections 1 to 7 of
Part II. It is therefore, worthwhile to enumerate the seven categories referred
to in Part

II. They are as under:-Sec. 1. Copyright and Related Right (Arts. 9-14)

2. Trade Mark (Arts. 15-21)

25 

 
 

3. Geographical Indications (Arts. 22-24)

4. Industrial Design (Arts. 25-26)

5. Patent (Arts. 27-34)

6. Layout-designs of Integrated Circuits (Arts. 35-38)

7. Protection of Undisclosed information (Art. 39)

8. Art. 40 Deals with control of anti-competitive practices in contractual


licenses.

Detailed Evaluation of TRIPs Agreement

So far, we have given a birds' eye view of TRIPs agreement. We shall now
discuss some important provisions of this agreement.

(a) Obligation of member states

(i) Members shall give effect to provisions of this agreement. Members may, but
shall not be obliged to implement in their domestic law more extensive protection
than is required by this Agreement, provided that such protection does not
contravene the provisions of this Agreement. Members shall be free to determine
the appropriate methods of implanting the provisions of this Agreement within
their own legal system and practice.

(ii) For the purpose of this AgreeI'1ent, the terms "intellectual property" refers to
all categories of intellectual property that are the subject of Sections 1 to 7 of Part
II.

(iii) Members shall accord the treatment provided for in this Agreement to the
nationals of other Members. In respect of the relevant intellectual property right,
the nationals of other Members shall be understood as those natural or legal
persons that would meet the criteria for eligibility for protection provided for the
Paris Convention (1967), the Berne Convention (1971), the Rome Convention and
the Treaty on Intellectual Property in Respect of Integrated Circuits, all Members
of the MTO members of those conventions. Any Member availing itself of the
26 

 
 

possibilities provided in paragraph 3 of Article 5 or paragraph 2 of Article 6 of the


Rome Convention shall make a notification as foreseen in those provisions to the
Council for Trade Related Aspects of Intellectual Property Rights.

TRIPs and other IPR Conventions

(i) In respect of Parts II, III and IV of this Agreement, members shall comply with
Articles 1-12 and 19 of the Paris Convention (1967).

(ii) Nothing in Parts I to IV of this Agreement shall derogate from existing


obligations that members may have to each other under the Paris Convention, the
Rome Convention and the Treaty on Intellectual Property in Respect of Integrated
Circuits.

(c) National Treatment

(i) Each member shall accord to the nationals of other member, treatment no less
favorable than that it accords to its own nationals with regard to the protection of
intellectual property, subject to the exceptions already provided in the Paris
Convention (1967), Berne Convention (1971), the Rome Convention and the
Treaty on Intellectual Property in Respect of Integrated Circuits, respectively.

(ii) Any member availing itself of the possibilities provided in Article 6 of the
Berne Convention and paragraph 1 (b) of Article 16 of The Rome Convention shall
make a notification as foreseen in those provisions to the Council for Trade
Related Aspects of Intellectual Property Rights.

(iii) Members may avail themselves of the exceptions permitted under paragraph 1
above in relation to judicial and administrative procedures, including the
designation of an address for service or the appointment of an agent within the
jurisdiction of a member only where such exceptions are necessary to secure
compliance with laws and regulations which are not inconsistent with the
provisions of this Agreement and where such practices are not applied in a manner
which would constitute a disguised restriction on trade.

27 

 
 

Nature and Scope of Protection available under TRIPs Agreement

As mentioned above there are 7 categories of intellectual properties mentioned in


Part II. The scope and nature of protection required to be given by each member
state may be discussed as follows;-

i)Copyright and other related rights (Arts. 9• 14)

The copyright protection shall extend to expression and not to ideas, procedures
and methods of operation or mathematical concepts as such. The member states
shall comply with certain provisions of Berne Convention. Computer
programmers, whether in source or object, Code shall be protected as literary book
under the Berne Convention (1971).

Similarly the producers of phonogram and broadcasting organization will have


exclusive rights and enjoy right to prohibit reproduction of their programmer,
previous; unauthorized fixation, broadcast or reproduction of live performance.
The terms of protection shall be for a minimum period of 50 years

(ii)Trade Mark (Arts. 15•21)

The owner of registered trade mark all have the exclusive right to prevent all third
parties not having his consent from using identical or similar signs for goods or
services. However his agreement also recognizes right available on the basis of
prior use. The term of trade mark shall be for an initial term of 7 years and may be
renewed indefinitely.

(iii)Geographical Indications (Arts. 22•24)

In respect of ~graphical indications, the agreement lays down that all parties must
provide means to prevent the use of any indication which misleads the consumer as
to the origin of goods, and any use which would constitute an act of unfair
competition. A higher level of protection is provided for geographical indications
for wines and spirits, which are protected

28 

 
 

Even where there is no danger of the public being misled as to the true origin.
Exceptions are allowed for names that have already become generic terms, but any
country using such as exception must be willing to negotiate a view to protecting
the geographical indications in question.

(iv)Industrial Design (Arts. 25-26)

Industrial designs are also protected under the agreement for a period of 10 years.
Owners of protected designs would be able to prevent the manufacture, sale or
importation of articles bearing or embodying a design which is a copy of the
protected design.

(v)Patent (Arts. 27-34)

As regards patents, there is a general obligation to comply with the substantive


provisions of the Paris Convention (1967). In addition, the agreement requires that
20 years patent protection be available for all inventions, whether of products or
process, in almost all fields of technology. Inventions may be excluded from
patentability if their commercial exploitation is prohibited for reasons of public
order or morally; otherwise, the permitted exclusions are for diagnostic,
therapeutic and surgical methods, and for plants and (other than microorganisms)
animals and essentially biological processes for the production of plants and
animals (other than micro-biological processes). Plant varieties, however, must be
protectable either by patents or by a sui generis system (such as the breeder's rights
provided in a UPOV Convention).

Detailed conditions are laid down for compulsory licensing or governmental use of
patents without the authorization of the patent owner. Rights conferred in respect
of patents for processes must extend to the products directly obtained by the
process. Under certain conditions alleged infringers may be ordered by a court to
prove that they have not used the patented process.

(vi) Layout Designs (Arts. 35-38)

29 

 
 

With respect to the protection of layout designs of integrated circuits, the


agreement requires parties to provide protection on the basis of the Washington
Treaty on Intellectual Property in Respect of Integrated Circuits which was opened
for signature in May 1989, but with a number of additions. Protection must be
available for a minimum period of 10 years.

(vii) Protection of undisclosed information (Trade Secrets) (Art. 39)

Trade secrets and know-how, which have commercial value, must be protected
against breach of confidentiality and other acts contrary to honest commercial
practices. The member states are required to protect trade secrets in respect to
pharmaceutical, agriculture and chemical products.

Enforcement of IPRs

Part III of the TRIPs on "Enforcement of Intellectual Property Rights" contains a


total of 21 Articles (Arts. 41-61), divided into five sections; General Obligations
(Art. 41); Civil and Administrative Procedures and Remedies (Arts. 42-49);
Provisional Measures (Art. 50) Special Requirements Related to Border Measures
(Arts. 51-60); and Criminal Procedures (Art. 61). Together these provisions are
aimed at first, on the measures that the holders of IPRs may require from the
judicial, administrative and customs authorities to take against infringements and
threatened infringements of their rights, and second, the procedures by which such
measures may be obtained. These measures may be civil as well as criminal in
nature to terminate and restrain infringements of IPRs.

Dispute Settlement System

The Dispute Settlement Body (DSB) is one of the most powerful yet controversial
bodies in WTO. It is a central element in providing security and predictability to
multilateral trading system. In fact, prompt, just and fair settlement of trade dispute
is essential to the effective functioning of WTO. The DSB is empowered to
establish panels, constituent appellate body, and adopt panel reports and exercise
surveillance and compliance with its decisions. The DSB is empowered to
establish panels, constituent appellate body, and adopt panel reports and exercise
surveillance and compliance with its decisions.
30 

 
 

Different Stages of Dispute Settlement

There are mainly six stages involved in settling disputes. Each stage has its
elaborate procedure and fixed time frame. They are:

Consultation

The first stage of settling dispute is holding consultation between disputing states.
Consultation aims to provide solution, which is mutually agreed. It is confidential
and does not prejudice the right of members to further proceedings. The process of
consultation should begin within 30 days of the request and should be completed
within 60 days.

Establishment of Panel

If consultation process fails to bring any solution within 60 days, the complainant
can ask the DSB to establish a panel. It consists of 3 to 5 experts. In case of dispute
between developed and developing countries one of the panel experts from
developing countries should be included. These panelists serve in their individual
capacity and not in their official position.

Panel Examination

The panel chosen in consultation with the parties examines the complaint. The
panel should complete its exercise within six months and give its final report
containing findings and recommendations. However, in case of urgency, the time
for giving final report can be reduced to three months. But in no case should the
period from establishment of the panel to the submission of the final report exceed
nine months.

Adoption of Panel Report

31 

 
 

The DSB adopts the panel report 60 days after it is issued. However, the DSB can
invite objection from the disputing members and can consider them before finally
adopting the report.

Appellate Review

Any party to the dispute can file an appeal against panel report before the
Appellate Body. A Standing Appellate Body is set up by the DSB to hear all
appeals. The Appellate Body is composed of seven persons who are representative
of WTO membership. The appeal proceedings are to be completed within 60 days
but in no case they shall exceed 90 days.

Implementation

It is the final stage of settlement system. Effective and proper implementation of


the ruling of DSB is necessary for the satisfactory resolution of the dispute. Thirty
days after the adoption of the panel or Appellate Body report, the party concerned
must inform the DSB of its intention with regard to implementation of the ruling.
In case of difficulty the member can be granted a reasonable time to implement the
ruling. Failure to implement the ruling within granted time can result into
suspension of concession. State found guilty of breach can also settle the claim by
paying compensation.

Q.4. what objectives the world intellectual property organization (WIPO) was
established?

Ans-Establishment of WIPO- is a landmark event in the history of promotion and


protection of intellectual property rights at global level. The convention
establishing the WIPO was signed at Stockholm on 14th July, 1967. The WI PO
has been designated as specialized body of United Nations and it also acts as a
complimentary body with WTO in so far as promotion and protection of IPRs is
concerned. The Head Quarter of WIPO is at Geneva.

32 

 
 

Objectives of WIPO-

1. To promote the protection of intellectual property throughout the world

Through co-operation among states and where appropriate in collaboration

With any international organization;

2. to harmonies national intellectual property legislation and procedures;

3. Provide services for international applications for intellectual property

Rights.

4. Exchange information on intellectual property;

5. Provide legal and technical assistance to developing and other countries;

6. Facilitate the resolution of private intellectual property disputes; and

7. Marshal information technology as a tool for storing, accessing and using


valuable intellectual property information.

Membership of WIPO-

Membership of WI PO is open to any state which is member of any Unions. Even


if it is not a member of Union, a nation can become member of WI PO if it is a
member of United Nations or other specialized agencies. The membership of
WIPO has gone up to 177 countries.

Organs of WIPO-

WIPO performs its functions and programmers through four organs-

General Assembly-

Consisting of the state’s party to the convention establishing WIPO which are the
members of the Unions;

33 

 
 

Conference-

Consisting of the state’s party to the Convention, whether or not they are members
of any of the Union;

Coordination Committee-

Consisting of the state’s party to the Convention which are members of the
Executive Committee of the Paris Union, or of the Berne Union or both; and

International Bureau-

It is the Secretariat of the Organization. It shall be directed by the Director


General, assisted by two or more deputy Directors General. The Director General
is the Chief Executive of the WIPO and he is appointed for a fixed term which is
not less than six years. The DG represents the organization, reports to and
conforms to the instructions of the General Assembly as to the internal and
external affairs of Organization.

Apart from these four organs the WIPO has established recently a World Wide
Academy for dissemination of IPR related information and knowledge throughout
the world.

Programmes and activities of WIPO-

1. Advice and expertise in the revision of national legislation is particularly


important for that WI PO.

2. member States with obligations under the TRIPs Agreement;

3. comprehensive education and training programmers at national and regional


levels for officials dealing with intellectual property, including those concerned
with enforcement and; for traditional and new groups of users on the value of
intellectual property and how to create their own economic assets through better
use of the intellectual property system

4. extensive computerization assistance to help developing countries acquire the


information technology resources (both in human and material terms) to streamline
34 

 
 

administrative procedures for managing and administering their own intellectual


property resources, and to participate in WI PO, global information network;

5. Financial assistance to facilitate participation in WIPO activities and meetings,


especially those.

6. Concerned with the progressive development of new international norms and


practices.

Special Programmes-

WIPO carries out a special programme to promote, in the developing countries, the
collective management of copyright, for, the increased benefit of creators such as
authors, composers and artists. A special unit looks after the concerns of the
least-developed countries, which require special attention and assistance so that
they too can reap the benefits of intellectual property.

Activities of WIPO-

A fundamental and enduring part of WIPO activities in promoting the protection of


intellectual property is the progressive development and application of
international norms and standards. The organization currently administers 11
treaties that set out internationally agreed rights and common standards for their
protection that the States which sign them agree to apply within their own
territories.

Development of Ecommerce-

In September 1999, WIPO adopted the Digital agenda-a work programme for the
Organization over the coming-years in response to the confluence of the internet
digital technologies and the intellectual property system. The Organization is
formulating appropriate responses that will encourage dissemination and use of
intellectual property such as music, films, trade identifiers and knowledge on the
internet as well as ensure protection of the rights of their creators and owners.

WIPO Worldwide Academy-

35 

 
 

WIPO believes that the human capital of developing countries is key to their
realizing the full benefits of the national and international intellectual property
system. Its activities include:-

1. Programmes Evolution of new training and teaching techniques.

2. Distance Learning Centre using Internet facilities.

3. Client specific learning modules and materials.

4. Use of modern public-access media to disseminate knowledge of

Intellectual property.

Inter-relationship between WIPO and WTO (World Trade Organization)-

Agreement between WIPO and WTO (which came into existence on 1st January,
1995) has been entered into force on 1st January, 1996 with a purpose of creating a
cooperative and conducive relationship. It inter alia provides:

1. Accessibility of laws and regulations in the WIPO collection by WTO

Members and their Nationals;

2. Accessibility of the computerized database to WTO members and their

Nationals; and.

3. Accessibility of Laws and Regulations in the WIPO collection by the

WTO Secretariat and the Council for TRIPs etc.

SHORT QUESTION:

Q.5.What was the objectives of universal copyright convention 1952?

Ans- The Universal Copyright Convention (UCC), adopted in Geneva, Switzerland,


in 1952, is one of the two principal international conventions protecting copyright;
the other is the Berne Convention.
36 

 
 

Two international protection regarding copyright

Berne convention 1886 universal copyright convention 1952

The UCC was developed by the United Nations Educational, Scientific and
Cultural Organization (UNESCO) as an alternative to the Berne Convention for
those states which disagreed with aspects of the Berne Convention, but still wished
to participate in some form of multilateral copyright protection. These states
included developing countries as well as the United States and most of Latin
America. The developing countries thought, that the strong copyright protections
granted by the Berne Convention overly benefited Western, developed,
copyright-exporting nations; whereas the United States and Latin America were
already members of the Buenos Aires Convention, a Pan-American copyright
convention that was weaker than the Berne Convention. The Berne Convention
states also became party to the UCC, so that their copyrights would exist in
non-Berne convention states. In 1973, the Soviet Union joined the UCC.

The United States only provided copyright protection for a fixed renewable term,
and required that in order for a work to be copyrighted, it must contain a copyright
notice and be registered at the Copyright Office. The Berne Convention, on the
other hand, provided for copyright protection for a single term based on the life of
the author, and did not require registration or the inclusion of a copyright notice for
copyright to exist. Thus, the United States would have to make several major
modifications to its copyright law in order to become a party to it. At the time, the
United States was unwilling to do so. The UCC thus permits those states, which
had a system of protection similar to the United States for fixed terms at the time
of signature, to retain them. Eventually, the United States became willing to
participate in the Berne convention, and change its national copyright law as
required. In 1989, it became a party to the Berne Convention as a result of the
Berne Convention Implementation Act of 1988.
37 

 
 

Under the Second Protocol of the Universal Copyright Convention (Paris text),
protection under U.S. copyright law is expressly required for works published by
the United Nations, by UN specialized agencies and by the Organization of
American States (OAS).[1] The same requirement applies to other contracting
states as well.

Berne Convention states were concerned, that the existence of the UCC would
encourage parties to the Berne Convention to leave that convention and adopt the
UCC instead. So the UCC included a clause stating, that parties, which were also
Berne Convention parties, need not apply the provisions of the Convention to any
former Berne Convention state which renounced the Berne Convention after 1951.
Thus, any state which has once adopted the Berne Convention, is penalised, if such
a state then decides to renounce the Berne Convention, and decides to use the UCC
protections instead, as its copyrights might no longer exist in Berne Convention
states. Since almost all countries are either members or aspiring members of the
World Trade Organization (WTO), and are thus conforming to the Agreement on
Trade-Related Aspects of Intellectual Property Rights Agreement (TRIPS), UCC
has lost its significance.

Q.6. Nature and Scope of Protection available under TRIPs Agreement?

As mentioned above there are 7 categories of intellectual properties mentioned in


Part II. The scope and nature of protection required to be given by each member
state may be discussed as follows;-

(i) Copyright and other related rights (Arts. 9• 14)

The copyright protection shall extend to expression and not to ideas,


procedures, and methods of operation or mathematical concepts as such.
The member states shall comply with certain provisions of Berne
Convention. Computer programmes, whether in source or object, Code
shall be protected as literary book under the Berne Convention (1971).

38 

 
 

Similarly the producers of phonogram and broadcasting organization will have


exclusive rights and enjoy right to prohibit reproduction of their programme,
previous; unauthorized fixation, broadcast or reproduction of live performance.
The terms of protection shall be for a minimum period of 50 years

(ii)Trade Mark (Arts. 15•21)

The owner of registered trade mark all have the exclusive right to prevent all third
parties not having his consent from using identical or similar signs for goods or
services. However his agreement also recognizes right available on the basis of
prior use. The term of trade mark shall be for an initial term of 7 years and may be
renewed indefinitely.

(ii) Geographical Indications (Arts. 22•24)

In respect of ~graphical indications, the agreement lays down that all parties must
provide means to prevent the use of any indication which misleads the consumer as
to the origin of goods, and any use which would constitute an act of unfair
competition. A higher level of protection is provided for geographical indications
for wines and spirits, which are protected

Even where there is no danger of the public being misled as to the true origin.
Exceptions are allowed for names that have already become generic terms, but any
country using such as exception must be willing to negotiate a view to protecting
the geographical indications in question. Furthermore, provision is made for
further negotiations to establish a multilateral system of notification and
registration of geographical indications for wines.

(iv)Industrial Design (Arts. 25-26)

Industrial designs are also protected under the agreement for a period of 10 years.
Owners of protected designs would be able to prevent the manufacture, sale or

39 

 
 

importation of articles bearing or embodying a design which is a copy of the


protected design.

(v)Patent (Arts. 27-34)

As regards patents, there is a general obligation to comply with the substantive


provisions of the Paris Convention (1967). In addition, the agreement requires that
20 years patent protection be available for all inventions, whether of products or
process, in almost all fields of technology. Inventions may be excluded from
patentability if their commercial exploitation is prohibited for reasons of public
order or morally; otherwise, the permitted exclusions are for diagnostic,
therapeutic and surgical methods, and for plants and (other than micro organisms)
animals and essentially biological processes for the production of plants and
animals (other than micro-biological processes). Plant varieties, however, must be
protectable either by patents or by a sui generis system (such as the breeder's rights
provided in a UPOV Convention).

Detailed conditions are laid down for compulsory licensing or governmental use of
patents without the authorization of the patent owner. Rights conferred in respect
of patents for processes must extend to the products directly obtained by the
process. Under certain conditions alleged infringers may be ordered by a court to
prove that they have not used the patented process.

(vi) Layout Designs (Arts. 35-38)

With respect to the protection of layout designs of integrated circuits, the


agreement requires parties to provide protection on the basis of the Washington
Treaty on Intellectual Property in Respect of Integrated Circuits which was opened
for signature in May 1989, but with a number of additions. Protection must be
available for a minimum period of 10 years.

Q.7. Different Stages of Dispute Settlement?

40 

 
 

There are mainly six stages involved in settling disputes. Each stage has its elaborate procedure
and fixed time frame. They are:

• Consultation
The first stage of settling dispute is holding consultation between disputing states. Consultation
aims to provide solution, which is mutually agreed. It is confidential and does not prejudice the
right of members to further proceedings. The process of consultation should begin within 30
days of the request and should be completed within 60 days.

• Establishment of Panel
If consultation process fails to bring any solution within 60 days, the complainant can
ask the DSB to establish a panel. It consists of 3 to 5 experts. In case of dispute between
developed and developing countries one of the panel experts from developing countries
should be included. These panelists serve in their individual capacity and not in their official
position.

• Panel Examination
The panel chosen in consultation with the parties examines the complaint. The panel should
complete its exercise within six months and give its final report containing findings and
recommendations. However, in case of urgency, the time for giving final report can be
reduced to three months. But in no case should the period from establishment of the panel to
the submission of the final report exceed nine months.
• Adoption of Panel Report
The DSB adopts the panel report 60 days after it is issued. However, the DSB can invite
objection from the disputing members and can consider them before finally adopting the
report.

• Appellate Review

41 

 
 

Any party to the dispute can file an appeal against panel report before the Appellate Body. A
Standing Appellate Body is set up by the DSB to hear all appeals. The Appellate Body is
composed of seven persons who are representative of WTO membership. The appeal
proceedings are to be completed within 60 days but in no case they shall exceed 90 days.

• Implementation
It is the final stage of settlement system. Effective and proper implementation of the ruling of
DSB is necessary for the satisfactory resolution of the dispute. Thirty days after the adoption
of the panel or Appellate Body report, the party concerned must inform the DSB of its
intention with regard to implementation of the ruling. In case of difficulty the member can be
granted a reasonable time to implement the ruling. Failure to implement the ruling within
granted time can result into suspension of concession. State found guilty of breach can also
settle the claim by paying compensation.

VERY SHORT QUESTION


Q.8 Inter-relationship between WIPO and WTO?

Agreement between WIPO and WTO (which came into existence on 1st January,
1995) has been entered into force on 1st January, 1996 with a purpose of creating a
cooperative and conducive relationship. It inter alia provides:

1. Accessibility of laws and regulations in the WIPO collection by WTO

Members and their Nationals;

2. Accessibility of the computerized database to WTO members and their

Nationals; and.

3. Accessibility of Laws and Regulations in the WIPO collection by the

WTO Secretariat and the Council for TRIPs etc.

Q.9. Define UNESCO?


42 

 
 

Ans--is a specialized agency of the United Nations (UN) based in Paris. Signed on 16 November
1945, came into force on 4 November 1946. Its declared purpose is to contribute to peace and
security by promoting international collaboration through educational, scientific, and cultural
reforms in order to increase universal respect for justice, the rule of law, and human rights along
with fundamental freedom proclaimed in the United Nations Charter.

It is the successor of the League of Nations' International Committee on Intellectual Cooperation.

UNESCO has 193 member states and 11 associate members.[3] Most of its field offices are
"cluster" offices covering three or more countries; national and regional offices also exist.

Q.10. Explain Paris convention,1883?

Ans- The Paris Convention applies to industrial property in the widest sense, including patents,
trademarks, industrial designs, utility models (a kind of "small-scale patent" provided for by the
laws of some countries), service marks, trade names (designations under which an industrial or
commercial activity is carried out), geographical indications (indications of source and
appellations of origin) and the repression of unfair competition.

The substantive provisions of the Convention fall into three main categories: national treatment,
right of priority, common rules.

Substantive provisions of convention-


ƒ National treatment
ƒ Right of priority
ƒ Commom rules

UNIT-IIIRD

43 

 
 

Q.1. what do you mean by patent right? What are its salient features? What
inventions are patentable and what are not? Discuss in details?

Ans- A patent can be defined as a grant of exclusive rights to an inventor over his
invention for a limited period of time. The exclusive rights conferred include the
right to make, use, exercise, sell or distribute the invention in India. The term of a
patent is twenty years, after the expiry of which, the invention would fall into the
public domain.

History of patent law-

A patent is an exclusive monopoly grant by the govt. of an inventor over his


invention for limited period of time some other types of intellectual property rights
are also called patents in some jurisdictions: industrial design rights are called
design patents in the US, plant breeders rights are sometimes called plant patents.
In modern usage, the term patent usually refers to the right granted to anyone who
invents any new, useful, and non-obvious process,machine, article of manufacture,
or composition of matter.The word patent originates from the Latin pat ere, which
means "to lay open" (i.e., to make available for public inspection).

Patents Act, 1970-

The Patents Act, 1970 remained in force for about 24 years without any change till
December 1994. Being a landmark in the industrial development of India, the basic
philosophy of the Act is that patents are granted to encourage inventions and to
secure that these inventions are worked on a commercial scale without undue
delay; and patents are granted not merely to enable patentee to enjoy a monopoly
for the importation of the patented article into the country. The said philosophy is
being implemented through compulsory licensing, registration of only process
patents for food, medicine or drug, pesticides and substances produced by chemical
processes which, apart from chemical substances normally understood, also
include items such as alloys, optical glass, semi- conductors, inter metallic
compounds etc. It may, however, be noted that products vital for our economy
such as agriculture & horticulture products, atomic energy inventions and all
living things are not patentable. Thus, the Patents Act 1970 was expected to
44 

 
 

provide a reasonable balance between adequate and effective protection of patents


on the one hand and the technology development, public interest and specific needs
of the country on the other hand. Uruguay round of GATT negotiations paved the
way for WTO. Therefore, India was put under the contractual obligation to amend
its Patents Act in compliance with the provisions of TRIPS. India had to meet the
first set of requirements on 1-1-1995.Accordingly an Ordinance effecting certain
changes in the Act was issued on 31st December 1994, which ceased to operate
after six months. Subsequently, another Ordinance was issued in 1999. This
Ordinance was subsequently replaced by the Patents (Amendment) Act, 1999 that
was brought into force retrospectively from 1st January, 1995.The amended Act
provided for filing of applications for product patents in the areas of drugs,
pharmaceuticals and agro chemicals though such patents were not allowed.
However, such applications were to be examined only after 31-12-2004.
Meanwhile, the applicants could be allowed Exclusive Marketing Rights (EMR) to
sell or distribute these products in India, subject to fulfilment of certain conditions.
India amended its Patents Act again in 2002 through the Patents (Amendment) Act,
2002 increasing the term of patent to 20 years for all technology, Reversal of
burden of proof, compulsory licenses etc. This Act came into force on 20th May
2003 with the introduction of the new Patent Rules, 2003 by replacing the earlier
Patents Rules, 1972.

SALIENT FEATURES OF THE ACT-

A patent is an exclusive right granted by a country to the owner of an invention to


make, use, manufacture and market the invention, provided the invention satisfies
certain conditions stipulated in the law.

A patent is an official document given to an inventor by the government allowing


him to exclude anyone else from commercially exploiting his invention for a
limited period which is 20 years at present. As per the Supreme Court, the object of
Patent Law is to encourage scientific research, new technology and industrial
progress. Grant of exclusive privilege to own, use or sell the method or the product
patented for a limited period, stimulates new inventions of commercial utility. The
price of the grant of the monopoly is the disclosure of the invention at the Patent
45 

 
 

Office, which, after the expiry of the fixed period of the monopoly, passes into the
public domain [M/s Bishwanath Prasad v. Hindustan Metal Industries, AIR1982
SC 1444]. By granting an exclusive right, patents provide incentives to individuals,
offering them recognition for their creativity and material reward for their
marketable inventions. In return for the exclusive right, the inventor has to
adequately disclose the patented invention to the public, so that others can gain the
new knowledge and can further develop the technology. The disclosure of the
invention is thus an essential consideration in any patent granting procedure.

Kinds of patents-

a. Product Patents-Product Patents are claims on a product per se and, hence, they
cover all possible uses of the product. Thus a product patent gives monopoly rights
over a physical thing or product and over all possible uses of the product
irrespective of the method through which the thing is produced.

b. Process Patent-A process patent gives protection only to a specific process or


method through a thing is produced or a specific use of it. Therefore the process
patent may be sought and granted for any particular process patents are generally
applicable in an atmosphere where the product is question is already known, or a
few methods of its production are known, or some uses person can claim is any
particular process or method of its production or any specific use of it.

c. Hybrid patents-A hybrid patent is one which is in between a product a product


and a process patent. In such a patent “the scope of protection is defined in term s
of the steps by which the product is produced.”

d. Secret Patents-If the government is of the opinion that the applicant is seeking a
grant of patent for an invention which is important with view point of safety or
defence of the country, the government may direct the person authorised to grant
patents, to keep the application and all documents pertinent thereto as a such until
after such time as the government deems fit. Such patent are called secret patents,
An eminent scholar describes the process thus: Once direction for secrecy are
given the application proceeds through the normal examination stages up to

46 

 
 

acceptance, but a patent granted, until secrecy is removed. If and when secrecy is
removed, the application proceeds normally to grant.

e. Petty Patents-As we have seen there may be a patent for a process or a product.
However, if the application of some advanced technology results in some other
form of that patented product which is useful in some way or the other, one can
get, in some countries, a patent for the said new configuration or design of the
patented product. Since the term for such a protection is smaller than an
independent patent, such protections are often termed as petty patents.

f. Dependent Patents-A patent which cannot be worked unless infringing an


existing patent needs a license from the patent office to use the existing patent. The
patent office may grant the said license, independently. However, if the proprietor
of the existing patent also wants a license to use the latter patent, the patent office
shall grant a similar license for him to use the latter patent. Such a patent which
cannot be exercised independently, and needs the use of an earlier patent for it (that
is, the former’s) successful working is known as a dependent patent.

International protection of patents-

• Paris convention on industrial property (20 march 1883)

• Patent cooperation treaty

• Rio convention on biological diversity

• Trips agreement

Criteria of patentability-The British Patent Act 1977 does not define the term
“invention” but provides that a patent may be granted for an invention if-

A. The invention is new;

B. The involves an inventive step;

C. It is capable of industrial application; and


47 

 
 

D. It is not declared by the Act as not patentable.

The Indian Patent Act defines the Term ‘invention’ as a new product or process
involving an inventive step and capable industrial application sec 2(1) (j) Indian
Patent Act. It further says that by an inventive step is meant ‘a feature of an
invention that involves technical advance as compared to the existing knowledge
or having economic significance or both and that makes the invention not obvious
to person skilled in the art sec 2(1) (j a) Indian Patent Act. It is also defines the
term “new invention” as any invention or technology .sec 2 (1) (1) Indian Patent
Act. Sec 2(d) provides that does not recognize incremental innovation of known
substance which does not increase the efficacy of the said known substance, is not
patentable. The intention of the legislature appears to be discouraging the efforts to
stretch the patent protection longer on the basis of cosmetic improvements made
upon the existing invention: “Thus provision does not recognize incremental
innovation of known substance. The objective of the section is to prevent drug
companies from unjustifiability prolonging the life of a patent by claiming new
properties. However, except these definitions the Indian Patents Act does not seem
to move any further. As a result it is not possible to find in a systematic and clear
way either the concept of Patents or criteria of a valid patent anywhere in the Act
country to the British Patents Act 1977 wherein such things have been provided in
all clarity. Therefore, we shall make a reference or two of the relevant provisions
of the UK Act to substantiate and enrich our discussion on patentability criteria.
Going through the clear cut provision of sec 1(1) of UK Patents Act definitions
given under sec 2 of Indian Patent Act, we find that the following criteria should
be fulfilled by an invention in order to get a grant of patent for it.

Novelty

A novel invention is one, which has not been disclosed, in the prior art where prior
art means everything that has been published, presented or otherwise disclosed to
the public on the date of patent (The prior art includes documents in foreign
languages disclosed in any format in any country of the world.

48 

 
 

(a) An invention shall not be considered to be novel if it has been anticipated by


publication before the date of the filing of the application in any of the
specification filed in pursuance of application for patent in India on or after the 1st
day of January 1912.

(b) An invention shall not be considered to be novel if it has been anticipated by


publication made before the date of filing of the application in any of the
documents in any country.

(c) An invention shall not be considered to be novel if it has been claimed in any
claim of any other complete specification filed in India which is filed before the
application but published after said application.

(d) An invention shall not be considered to be novel if it has been anticipated


having regard to the knowledge, oral or otherwise, available within any local or
indigenous community in India or elsewhere. In Ganendro Nath Banerji v. Dhanpal
Das Gupta, AIR1945 Oudh 6, it was held that no general rule can be laid down as
to what does or does not constitute an invention. The general criterion seems to be
whether that which is claimed lies within the limits of development of some
existing trade, in the sense that it is such a development as an ordinary person
skilled in that trade could, if he wishes so to do, naturally, make without any
inventive step. But novelty need only be established in the process of
manufacturing, not in the article produced. Novel combination of two known ideas
may be sufficient to establish novelty of subject matter in this respect. In Ram
Narain Kher v. M/s Ambassador Industries, AIR 1976 Del 87, the Delhi High
Court has held that at the time the patent is granted to a party it is essential that the
party claiming patent should specify what particular features of his device
distinguish it from those which had gone before and show the nature of the
improvement which is said to constitute the invention. A person claiming a patent
has not only to allege the improvement in art in the form but also that the
improvement effected a new and very useful addition to the existing state of
knowledge. The novelty or the invention has to be succinctly stated in the claim.

Case-Cloth workers of Ipswich case,Darcy v/s allen)

49 

 
 

Inventive Step-

Inventive step is a feature of an invention that involves technical advance as


compared to existing knowledge or having economic significance or both, making
the invention non obvious to a person skilled in art. Here definition of inventive
step has been enlarged to include economic significance of the invention apart
from already existing criteria for determining inventive step. An invention shall not
be considered as involving an inventive step, if, having regard to the state of the
art, it is obvious to a person skilled in the art. The term "obvious"
means that which does not go beyond the normal progress of technology but
merely follows plainly or logically from the prior art, i.e. something which does
not involve the exercise of any skill or ability beyond that to be expected of the
person skilled in the Art. For this purpose a “person skilled in the art” should be
presumed to be an ordinary practitioner aware of what was general common
knowledge in the relevant art at the relevant date. In some cases the person skilled
in the art may be thought of as a group or team of persons rather than as a single
person.

Case-(harword v/s great northern railway co)

Industrial Applicability- An invention is capable of industrial application if it


satisfies three conditions, cumulatively:

a. can be made;

b. can be used in at least one field of activity;

c. can be reproduced with the same characteristics as many times as necessary.

1. An invention to be patentable must be useful. If the subject matter is devoid of


utility it does not satisfy the requirement of invention.

2. For the purpose of utility, the element of commercial or pecuniary success has
no relation to the question of utility in patent law.

50 

 
 

3. The usefulness of an alleged invention depends not on whether by following the


directions in the complete specification all the results not necessary for commercial
success can be obtained, but on whether by such directions the effects that the
application/patentee professed to produce could be obtained.

4. The meaning of usefulness is therefore useful for the purpose indicated by the
applicant or patentee whether a non-commercial utility is involved.

5. The usefulness of the invention is to be judged, by the reference to the state of


things at the date of filing of the patent application, if the invention was then
useful, the fact that subsequent improvement have replaced the patented invention
render it obsolete and commercially of no value, does not invalidate the patent.

7. Speculation or imaginary industrial uses are not considered to satisfy the


industrial application requirement.

Case-(Darcy v/s allein1602)

INVENTIONS NOT PATENTABLE-

Under section 3 of the Patents Act, 1970, the following are not inventions and
hence are not considered to be patentable. However, examples given are mere
illustrations and may not be conclusive on the subject. Objective decisions may be
taken on case to case basis.

1. The mere discovery of a scientific principle or the formulation of an abstract


theory or discovery of any living thing or non-living substance occurring in nature
is not an invention:

(i) A claim for discovery of scientific principle is not considered to be an


invention, but such a principle when used with a process of manufacture resulting
in a substance or an article may be considered to be an invention.

(ii) A scientific theory is a statement about the natural world. These theories
themselves are not considered to be inventions, no matter how radical or
revolutionary an insight they may provide, since they do not result in a product or
process. However, if the theory leads to practical application in the process of
51 

 
 

manufacture of an article or substance, it may well be patentable. A claim for


formulation of abstract theory is not considered to be an invention. For example,
the fact that a known material or article is found to have a hitherto unknown
property is a discovery and not an invention. But if the discovery leads to the
conclusion that the material can be used for making a particular article or in a
particular process, then the article or process could be considered to be an
invention.

A method of agriculture or horticulture is not an invention. Examples of subject


matters excluded from patentability under this provision are:

(a) A method of producing a plant, even if it involved a modification of the


conditions under which natural phenomena would pursue their inevitable course
(for instance a greenhouse).

(b) A method of producing improved soil from the soil with nematodes by treating
the soil with a preparation containing specified Phosphorathioates.

(c) A method of producing mushrooms.

(d) A method for cultivation of algae.

Any process for the medicinal, surgical, curative, prophylactic, diagnostic,


therapeutic or other treatment of human beings or any process for a similar
treatment of animals to render them free of disease or to increase their economic
value or that of their products is not an invention. This provision excludes from
patentability, the following:

(a) Medicinal methods: As for example a process of administering medicines


orally, or through injectable, or topically or through a dermal patch.

(b) Surgical methods: As for example a stitch-free incision for cataract removal.

(c) Curative methods: As for example a method of cleaning plaque from teeth.

(d) Prophylactic methods: As for example a method of vaccination.

52 

 
 

(e) Diagnostic methods: Diagnosis is the identification of the nature of a medical


illness, usually by investigating its history and symptoms and by applying tests.
Determination of the general physical state of an individual (e.g. a fitness test) is
considered to be diagnostic.

(f) Therapeutic methods: The term ―”therapy” includes prevention as well as


treatment or cure of disease. Therefore, the process relating to therapy may be
considered as a method of treatment and as such not patentable.

(g) Any method of treatment of animal to render them free of disease or to increase
their economic value or that of their products for example a method of treating
sheep for increasing wool yield or a method of artificially inducing the body mass
of poultry.

(h) Further examples of subject matters excluded under this provision are: any
operation on the body, which requires the skill and knowledge of a surgeon and
includes treatments such as cosmetic treatment, the termination of pregnancy,
castration, sterilization, artificial insemination, embryo transplants, treatments for
experimental and research purposes and the removal of organs, skin or bone
marrow from a living donor, any therapy or diagnosis practiced on the human or
animal body

(iii) Finding out that a particular known material is able to withstand mechanical
shock is a discovery and therefore not patentable, but a claim to a railway sleeper
made of the material would not fall foul of this exclusion, and would be allowable
if it passed the tests for novelty and inventive step. Similarly, finding of a new
substance or micro-organism occurring freely in nature is a discovery and not an
invention. And further includes methods of abortion, induction of labour, or
menstrual regulation.

(i) Application of substances to the body for purely cosmetic purposes is not
therapy.

(j) Patent may however be obtained for surgical, therapeutic or diagnostic


instrument or apparatus. Also the manufacture of prostheses or artificial limbs and
taking measurements thereof on the human body are patentable.
53 

 
 

Plants and animals in whole or any part thereof other than micro-organisms but
including seeds, varieties and species and essentially biological processes for
production or propagation of plants and animals are not inventions.

The subject matters excluded under this provision are:

(a) Plants in whole or in part

(b) Animals in whole or in part

(c) Seeds

(d) Varieties and species of plants and animals

(e) Essentially biological processes for production or propagation of plants and

animals. Microorganisms, other than the ones discovered from the nature, may be
patentable. For instance, genetically modified microorganisms may be patentable
subject to other requirements of Patentability. A new process of preparation of a
vaccine under specific scientific conditions, the vaccine useful for protecting
poultry against contagious bursitis infection was held to be patentable by the Court
on the ratio that the statute does not make a manner of manufacture as
un-patentable even if the end products contains a living organism. ( Dimminaco
A.G. Vs. Controller of Patents & Designs and Others) Plant varieties are
provided protection in India under the provisions of the Protection of Plant
Varieties and Farmers Rights Act, 2002.

Q.2. Define patent in bio technology?

Ans-Biotechnology today plays a key role in virtually all areas of


pharmaceutical science, genetics, molecular biology, biochemistry,
immunology, stem cell related research embryology and cell biology,
bioremediation and biodegradation.

54 

 
 

The development of the genetic resources of biodiversity is known as


biotechnology. Broadly defined, biotechnology includes any technique that uses
living organisms or parts of organisms to make or modify products, to improve
plants or animals, or to develop microorganisms for specific uses (Congress of
the United States, Office of Technology Assessment, 1990). Mankind has used
forms of biotechnology since the dawn of civilization. However, it has been the
recent development of new biological techniques (e.g., recombinant DNA, cell
fusion, and monoclonal antibody technology) which has raised fundamental
social and moral questions and created problems in intellectual property rights.
Apart from Pharmaceutical sectors, the biotechnology innovations and research
are instrumental in health care systems, agricultural industry, polymers &
materials sectors, etc.In order to help the patent seekers, a Biotechnology Patent
Facilitation Cell (BPFC)2 was established by the Department of Biotechnology
(DBT) in July'1999. BPFC has been catering to the need of promotion of biotech
research by:
1.Creating awareness and understanding among biologists and biotechnologists,
relating to patents and the challenges and opportunities in this area
2. Providing patenting facilities to biologists and biotechnologists in the country
for filing Indian and foreign patents on a sustained basis.
3. Keeping a watch on development in the area of IPR and make important
issues known to policymakers, bio-scientists, biotech industry, etc.
Another government authority working for the same cause is the Council of
Scientific and Industrial research (CSIR) which has moved from their earlier
mantra of "publish or perish" to "patent or perish". The Indian Government has
under its "Science and Technology Policy-2003" highlighted below aspects:

55 

 
 

4. Science and technology governance and investments


5. Optimal utilization of existing infrastructure and competence by
networking of existing infrastructure.
6. Strengthening of infrastructure of Science and Technology in academic
institutions.
7. New funding mechanism for basic research
8. Human resource development.
9. Technology development, transfer and diffusion
10. Promotion of innovation.
11. Autonomous Technology Transfer organizations would be created in
academic institutes to facilitate transfer of know-how generated in industry.
12. Indigenous resources and traditional knowledge: Development of
technologies that add value to India's indigenous resources would be supported
and the Indian share in the global herbal product market would be increased.
13. Technologies for mitigation and management of natural hazards
14. Generation and management of intellectual property: The fullest protection
to competitive intellectual property from Indian R&D programs would be made.
15. Public awareness of science and technology
16. International science and technology cooperation:
Those international collaborative programs contributing directly to India's
scientific development and security objectives would be courage.
17. Fiscal measures: Innovative fiscal measures are planned and strategies for
attracting higher levels of investments both public and private in science and
technological development.

56 

 
 

One of the significant sections of the Patents Act, 1970, which plays an
important role in the patenting of the inventions in the field of biotechnology
(more in the pharmaceutical sector) is Section 3(d) of the Patents Act, 1970. The
said sections excludes the below from being invention under the Act, the mere
discovery of new form of known substances which does not result in
enhancement of the known efficacy of that substance or the mere discovery of
any new property or new use for a known substances or the mere use of a known
process, machine or apparatus unless such known process, machine or apparatus
results in a new product or employs at least one new reactant. Hence while
proceeding with the patent application for the biotechnological innovation the
applicant might be called upon to establish that the applied invention has
enhancement in efficacy and difference in properties over the existing product/
substance.
Further Section 3(i) of the Indian patent Act excludes medical treatment
methods from being an invention. Section 3 (i) states that any process for the
medicinal, surgical, curative, prophylactic (diagnostic, therapeutic) or other
treatment of human beings or any process for a similar treatment of animals to
render them free of disease or to increase their economic value or that of their
products shall not be considered as invention and hence is not patentable.
THE GOVERNMENT POLICIES IN PATENTING-
Ministry of Science and Technology has issued the guidelines "Instructions for
Technology Transfer and Intellectual Property Rights". The salient features of
the guidelines are reproduced below from the official website:
1. Ownership of Intellectual Property: Institutions shall be encouraged to seek
protection of intellectual property rights in respect of the results of R&D. They

57 

 
 

may retain the ownership of such IPR. 'Institutions' would mean any technical,
scientific or academic establishment where research is carried out through
funding by the central/state government.
2. Transfer of Technology: The institutions shall take the necessary steps to
commercially exploit patents on exclusive or non-exclusive basis.
3. Royalty to Inventors: The owner institution is permitted to retain the
benefits and earnings generated out of the IPR. The institution may determine
the share of inventor(s) and other persons from such actual earnings. However,
such share(s) shall be limited to one third of the actual earnings.
4. Patent Facilitating Fund: The owner institution(s) shall set apart no less
than 25% of the revenue generated from IPR, to create a Patent Facilitating
Fund. The fund shall be utilized by the owner for updating inventions, filing
new patent applications and protecting the IP rights against infringement and for
building competency in the area of IPR and related issues.
5. Information: The institutions shall submit information relating to the details
of the patent obtained the benefits and earnings arising out of IPR and the
turnover of the products periodically to the department/Ministry which had
provided funds.
6. March in Rights: The Government shall have a royalty-free license for the
use of intellectual property for the purposes of the Government of India
LEGISLATIVE FRAMEWORK AND POLICIES FOR THE PROTECTION
OF BIOTECHNOLOGY-
• Environment Protection Act, 1986- The Environment Protection Act, 1986-
The Rules for the Manufacture, Use, Import, Export and Storage of Hazardous

58 

 
 

Microorganisms/ Genetically Engineered Organisms or Cells 1989 (Rules,


1989) were first notified under the Environment (Protection) Act, 1986.
• EXIM Policy- Units undertaking to export their entire production of goods
and services may be set up under Bio-Technology Park (BTP) scheme for
manufacture of goods.
• Recombinant DNA Safety Guidelines established by The Department of
Biotechnology- The guidelines cover areas of research involving genetically
engineered organism. It also deals with genetic transformation of green plants, r
DNA technology in vaccine development and on large scale production and
deliberate/ accidental release of organisms, plants, animals and products derived
by r DNA technology into the environment. The issues relating to Genetic
Engineering of human embryos, use of embryos and foetus in research and
human germ line gene therapy are excluded from the scope of the guidelines.
• Guidelines for Research in Transgenic Plants & Guidelines for Toxicity
and Allergenicity Evaluation of Transgenic Seeds, Plants and Plant Parts, 1998
were given by Department of Biotechnology, Ministry of Science and
Technology- The current guidelines cover areas of recombinant DNA research
on plants including the development of transgenic plants and their growth in soil
for molecular and field evaluation. The guidelines also deal with import and
shipment of genetically modified plants for research use only.
• National Seed Policy, 2002- This policy aims and objects as, biotechnology
will be a key factor in agricultural development in the coming decades.
• Seeds Act, 1966- This Act provides for compulsory registration of seed on
the basis of their performance, deregulation/ decontrol of seed industry/

59 

 
 

processing units and imposition of more stringent penalties to check the sale o
the spurious seeds.
• Drugs & Cosmetic Act 1940- An Act to regulate the import, manufacture,
distribution and sale of drugs and cosmetics.
• Drug Policy, 2002-
The main objectives of this policy are: -
a. Ensuring abundant availability at reasonable prices within the country of
good quality essential pharmaceuticals of mass consumption;
b. Strengthening the indigenous capability for cost effective quality
production and exports of pharmaceuticals by reducing barriers to trade in the
pharmaceutical sector;
c. Strengthening the system of quality control over drug and pharmaceutical
production and distribution to make quality an essential attribute of the Indian
pharmaceutical industry and promoting rational use of pharmaceuticals;
d. Encouraging R&D in the pharmaceutical sector in a manner compatible
with the country's needs and with particular focus on diseases endemic or
relevant to India by creating an environment conducive to channelizing a higher
level of investment into R&D in pharmaceuticals in India;
e. Creating an incentive framework for the pharmaceutical industry which
promotes new investment into pharmaceutical industry and encourages the
introduction of new technologies and new drugs;
• Biological Diversity Act, 2005- Provides for conservation of biological
diversity, sustainable sharing of the benefits arising out of the use of biological
resources, knowledge and related matters;

60 

 
 

• National Guidelines of Stem Cell Research and Therapy 2007 were


established by Department of Biotechnology and Indian Council of Medical
research -

The main objectives are:


a. To lay down general principles for stem cell research and therapy keeping
in view the ethical issues;
b. To formulate specific guidelines for derivation, propagation,
differentiation, characterization, banking, and use of human stem cells for
research and therapy;
• India is one of the first Asian countries to invest in agricultural
biotechnology research and to set up a bio-safety system to regulate the approval
of genetically modified (GM) crops. This provides the base for Agri-biotech
companies.

Q.3. Discuss the patent protection for computer programme?


Ans- INTRODUCTION-
The laws which govern the protection of computer software fall under the
domain of intellectual property. Intellectual property protection is generally
granted for the benefit of both creator of the property and public welfare. There
is a three step process linking the public welfare with intellectual property. The
first step involves expanding the scope of legal protection offered to software
creator by granting them enhanced monopoly rights. The second step is this kind
of enhanced protection creates a reward system motivating further creativity.
Finally, this expansion of inventive activity brings about the discovery of more

61 

 
 

ideas and faster advancement of technology. The end result of this process is
that the public receives different range of software products.
The granting of intellectual property protection to computer programmes can be
seen as a form of legal subsidization to a particular industry and technology. The
intellectual property regimes that protect computer software have had a direct
impact on the ownership and user regimes that have been established; the
alternatives to proprietary software, open source and free software have been a
philosophical and practical response to the existing legal regimes.
SOFTWARE PATENT HISTORY IN INDIA-
In India the 1970 patent Act exclude mathematical or business or a computer
program per se or algorithms from patentability. In the last 10 years of India’s
experiments with TRIPS compliance in the domain of patents, put a major test in
section.3.
In 2004 a major amendment was introduced in section 3 through Patent
Amendment Ordinance with respect to the patentability of computer programs.
By the new clause sec.3 (k), a computer program per se other than its technical
application to industry or a combination with hardware; a mathematical method
or business method or algorithms. The key expressions in the amendment are
“technical application to industry” and “combination with hardware”. This
means that if an invention is directed at computer software having technical
application to industry or coupled to hardware then it is patentable. But this
attempt was short lived as the government repealed ordinance through an
amendment Act in 2005.
This section laid down a blanket prohibition on the patentability of computer
software; it extends only to computer programs standing alone. So this clause

62 

 
 

would be interpreted to mean that an invention would be patentable only if a


computer program is one of its elements. If the invention as a whole includes
something more than the computer program that is eligible for patent protection
WHY PROTECT COMPUTER SOFTWARE-
“Computer software” also referred to as computer programs are the instructions
executed by a computer. In other words, the explanations, instructions,
commands and systems which have been developed in order to run the machine
are called „computer software‟. Software comprises of the following one or
more components: the source code itself which contains the programmer’s
invaluable comments any literature that may be supplied with the package which
could be in the form of manuals or explanatory material regarding the running of
the programmer. All these components require protection because the making of
it involves the expenditure of skill, time and labor and therefore the resultant
work should be protected from misappropriation.
Software has a market value. Computer software is subject to ferocious
competition with a shorter life circle and is liable to be copied soon, as it is
“read all on the face” technology. Because of its nature the owner of computer
program will have two problems (i) economic problem and (ii) competition.
Economic problem means, others can access it without payment to the creator.
Competition means the competitors will make competing products based on the
creation either by reserve engineering or blatant copying. Apart from protecting
the economic interest of the owner the protection of software through
appropriate IPR mechanism is considered necessary to encourage creativity,
innovation and investment. As already mentioned software may be reproduced

63 

 
 

at no cost, some means of restricting the free copying and redistribution of


software work is necessary to preserve an investment in a software product.
To establish intellectual property protection to computer software domestically
and internationally the signatories of TRIPS Agreement, Berne convention, and
WIPO Copyright Treaty (WCT) have agreed to protect copyright in a computer
program until, at a minimum 50 years after the author (software writer) of the
program dies. For citizens of more than 162 members of Berne convention
countries, once protection is grated to a work in one member country that work
is automatically protected within the borders of all other signatory countries.
However, it is the discretion of the states to given protection for computer
program under copyright or patent laws.
PATENT PROTECTION-
Unlike the copyright law which merely protects the expression of an idea,
patent law protects the concepts of the invention. Currently some countries
protect computer software like any other invention as long as it is a proper
subject for patent protection i.e. if it is a new and useful process involving an
inventive step and capable of industrial application. The subjects which
excluded from patent protection are laws of nature, natural phenomena, abstract
ideas and mathematical expressions of scientific truths. Mathematical and
scientific expressions are denied patent protection because technology is
suppressed against the desires of the authors of the constitution, if such patents
are granted.
Comparing to the protection given under patent law, the protection given by
copyright and trade secrets has limited scope. The owner of the copyright over
an item of software has the right to prevent any other person from copying the

64 

 
 

code as it is written but does not have the right to prevent the utilization of idea
behind the code, providing that the person utilizing the idea must use in a
manner that different from the arrangement of the code. The copyright law is
also limited to prohibit unauthorized copying of the protected work but it does
not prohibit all forms of copying. The expression of a method of operation and
principles of a computer program cannot be protected by copyright. Functional
aspects of a computer program are excluded from copying. A patent provides
more secure protection than the copyright and the trade secret. It protects the
„idea‟ or „functionality‟ of the software. Copying of an idea is very easy to do
and anybody can describe it simply, that is might a patent is restricting from
doing.
If a computer software is merely an algorithm it should not be protected under
patents. The term of algorithm is not defined in the patent act. If the invention is
technical in nature it will entitled to get protection under patents. The
mathematical algorithms which per se are not regarded as patentable subject
matter universally, they are merely considered as abstract ideas or mental steps.
ADVANTAGES OF PATENT PROTECTION-
Patenting software has three specific advantages over both copyright and trade
secret protection.
• If a patent holder files a patent infringement claim he cannot be faced with
the defense of independent creation.
• The patent grant the creator a monopoly right to license his product and
since disclosure of the invention is a requirement of patentability, the inventor
need not concern himself with the secrecy problems facing copyright and trade
secret holders.

65 

 
 

• The patent holder receives a 20 year monopoly over the invention, during
which time others are prohibited from making using or selling the invention.
There has not been any international convention which granting patent to
computer software. The Paris convention, 1883, on the protection of industrial
property, leaves it to member countries to specify the protectable subject matter
for patents.
The World Intellectual Property Organization (WIPO) 1978 provided that for
granting patent protection for computer software it must fulfill the criteria’s of
new and inventive technical solutions. However in most of the countries, the
question of patentability cannot be answered with any degree of certainty.
The Agreement on Trade Related Aspects of Intellectual Property Rights
(TRIPS) describes patentable subject matter under article 27.1, subjects to
certain exceptions or conditions under the agreement “patent shall be available
for inventions, whether products or processes, in all fields of technology,
provided they are new, involve an inventive step and are capable of industrial
application”. TRIPS prohibit any field specific exclusion of patents.

Q.4. Discuss the process of obtaining patent?


Ans- Step 1: Write down the invention (idea or concept) with as much details
as possible
Collect all the information about your invention such as:
• Area of invention
• Description of the invention what it does?
• How does it work?

66 

 
 

• Advantages of the invention


Ideally, if you have worked on the invention during research and development
phase you should have something call lab record duly signed with date by you
and respective authority.
Step 2: include drawings, diagrams or sketches explaining working of
invention
The drawings and diagrams should be designed so as to explain the working of
the invention in better way with visual illustrations. They play an important role
in patent application.
Step 3: check whether the invention is patentable subject matter
All inventions may not be patentable, as per Indian patent act there are certain
inventions that are not patentable explained in detail in (inventions not
patentable)
Step 4a: Patentability search
The next step would be finding out whether your invention meets all
patentability criteria as per Indian patent act? That is,
• Novelty
• Non-obviousness
• Industrial application
• Enabling
The detailed explanation for patentability criteria is given here (what are
patentability criteria’s). The patentability opinion is provided by the patent
professionals up on conducting extensive search and forming patentability
report.
Step 4b: Decide whether to go ahead with patent

67 

 
 

The patentability report and opinion helps you decide whether to go ahead with
the patent or not, chances are what you thought as novel might already been
patented or know to public in some form of information. Hence this reports
saves lots of time, efforts and cost of the inventor by helping him decide
whether to go ahead with the patent filing process or not.
Step 5: Draft (write) patent application
In case you are at very early stage in the research and development for your
invention, then you can go for provisional application. It gives following
benefits:
• Secures filing date
• 12 months of time to file complete specification
• Low cost
After filing provisional application, you secure the filing date which is very
crucial in patent world. You get 12 months of time to come up with the
complete specification, up on expiry of 12 months your patent application will
be abandoned.
When you complete the required documents and your research work is at level
where you can have prototype and experimental results to prove your inventive
step you can file complete specification with patent application.
Filing the provisional specification is the optional step, if you are at the stage
where you have complete information about your invention then you can
directly go for complete specification.
Step 6: Publication of the application
Up on filing the complete specification along with application for patent, the
application is published after 18 months of first filing.

68 

 
 

An early publication request can be made along with prescribed fees if you do
not wish to wait till the expiry of 18 months from the date of filing for
publishing your patent application.
Generally the patent application is published within a month form request form
early publication.
Step 7: Request for examination
The patent application is examined only after receiving request for examination
that is RFE. Up on receiving this request the controller gives your patent
application to a patent examiner who examinees the patent application with
different patentability criteria like:
• Patentable subject matter
• Novelty
• Non-obviousness
• Inventive step
• Industrial application
• Enabling
The examiner creates a first examination report of the patent application upon
reviewing it for above terms. This is called patent prosecution. Everything
happening to patent application before grant of patent is generally called as
patent prosecution.
The first examination report submitted to controller by examiner generally
contains prior arts (existing documents before the date of filing) which are
similar to the claimed invention, and same is reported to patent applicant.
Step 8: respond to objections

69 

 
 

Majority of patent applicants will receive some type of objections based on


examination report. The best thing to do it analyses the examination report with
patent professional (patent agent) and creating a response to the objections
raised in the examination report.
This is a chance for an inventor to communicate his novelty over prior arts
found in the examination report. The inventor and patent agent create and send a
response to the examination that tries to prove to controller that his invention is
indeed patentable and satisfies all patentability criteria’s.
Step 9: clearing all objections
This communication between controller and patent applicant is to ensure that all
objections raised in the patent application are resolved. (if not the patent will not
be granted ) and the inventor has his fair chance to prove his point and establish
novelty and inventive step over existing prior arts.
Up on finding the patent application in order of grant, it is grant to the patent
applicant as early as possible.
Step 10: Grant of patent
The application would be placed in order for grant once it is found to be
meeting all patentability requirements. The grant of patent is notified in the
patent journal which is published time to time.

70 

 
 

SHORT QUESTION:
Q.5. discuss the history of patents?
Ans-The first legislation in India relating to patents was the Act VI of 1856.
The objective of this legislation was to encourage inventions of new and useful
manufactures and to induce inventors to disclose secret of their inventions. The
Act was subsequently repealed by Act IX of 1857 since it had been enacted
without the approval of the British Crown. Fresh legislation for granting
‘exclusive privileges’ was introduced in 1859 as Act XV of 1859. This
legislation contained certain modifications of the earlier legislation, namely,
grant of exclusive privileges to useful inventions only and extension of priority
period from 6 months to 12 months. This Act excluded importers from the
definition of inventor. This Act was based on the United Kingdom Act of 1852
with certain departures which include allowing assignees to make application in
India and also taking prior public use or publication in India or United Kingdom
for the purpose of ascertaining novelty.
• In 1872, the Act of 1859 was consolidated to provide protection relating to
designs. It was renamed as “The Patterns and Designs Protection Act” under Act
XIII of 1872. The Act of 1872 was further amended in 1883 (XVI of 1883) to
introduce a provision to protect novelty of the invention. A grace period of 6
months was provided for filing such applications after the date of the opening of
such Exhibition.
• This Act remained in force for about 30 years without any change but in
the year 1883, certain modifications in the patent law were made in United
Kingdom and it was considered that those modifications should also be
incorporated in the Indian law. In 1888, an Act was introduced to consolidate

71 

 
 

and amend the law relating to invention and designs in conformity with the
amendments made in the U.K. law.
• The Indian Patents and Designs Act, 1911, (Act II of 1911) replaced all the
previous Acts. This Act brought patent administration under the management of
Controller of Patents for the first time. This Act was further amended in 1920 to
enter into reciprocal arrangements with UK and other countries for securing
priority. In 1930, further amendments were made to incorporate, inter-alia,
provisions relating to grant of secret patents, patent of addition, use of invention
by Government, powers of the Controller to rectify register of patent and
increase of term of the patent from 14 years to 16 years. In 1945, an amendment
was made to provide for filing of provisional specification and submission of
complete specification within nine months.
• After Independence, it was felt that the Indian Patents & Designs Act, 1911
was not fulfilling its objective. Accordingly, the Government of India
constituted a committee under the Chairmanship of Justice (Dr.) BakshiTilak
Chand, a retired Judge of Lahore High Court, in 1949 to review the patent law in
India in order to ensure that the patent system is conducive to the national
interest. The terms of reference included —
1) To survey and report on the working of the patent system in India;
2) To examine the existing patent legislation in India and to make
recommendations for improving it, particularly with reference to the provisions
concerned with the prevention of abuse of patent rights;
3) To consider whether any special restrictions should be imposed on patent
regarding food and medicine;

72 

 
 

4) To suggest steps for ensuring effective publicity to the patent system and to
patent literature, particularly as regards patents obtained by Indian inventors;
5) To consider the necessity and feasibility of setting up a National Patents
Trust;
6) To consider the desirability or otherwise of regulating the profession of
patent agents
7) To examine the working of the Patent Office and the services rendered by it
to the public and make suitable recommendations for improvement; and
8) To report generally on any improvement that the Committee thinks fit to
recommend for enabling the Indian Patent System to be more conducive to
national interest by encouraging invention and the commercial development and
use of inventions.
• The committee submitted its interim report on 4th August, 1949 with
recommendations for prevention of misuse or abuse of patent right in India and
suggested amendments to sections 22, 23 & 23A of the Patents & Designs Act,
1911 on the lines of the United Kingdom Acts 1919 and 1949.
• Based on the above recommendation of the Committee, the 1911 Act was
amended in 1950(Act XXXII of 1950) in relation to working of inventions and
compulsory license/revocation. Other provisions were related to endorsement of
the patent with the words ‘license of right’ on an application by the Government
so that the Controller could grant licenses. In 1952 (Act LXX of 1952).
• In 1957, the Government of India appointed Justice N. Rajagopala
Ayyangar Committee to examine the question of revision of the Patent Law and
advise government accordingly. The report of the Committee, which comprised
of two parts, was submitted in September, 1959. This report recommended

73 

 
 

major changes in the law which formed the basis of the introduction of the
Patents Bill, 1965. This bill was introduced in the LokSabha on 21st September,
1965, which however lapsed. In 1967, again an amended bill was introduced
which was referred to a Joint Parliamentary Committee and on the final
recommendation of the Committee, the Patents Act, 1970 was passed. This Act
repealed and replaced the 1911 Act so far as the patents law was concerned.
However, the 1911 Act continued to be applicable to designs. Most of the
provisions of the 1970 Act were brought into force on 20 th April 1972 with
publication of the Patent Rules, 1972.
• This Act remained in force for about 24 years without any change till
December 1994. An ordinance effecting certain changes in the Act was issued
on 31 st December 1994, which ceased to operate after six months.
Subsequently, another ordinance was issued in 1999. This ordinance was
subsequently replaced by the Patents (Amendment) Act, 1999 that was brought
into force retrospectively from 1st January, 1995. The amended Act provided for
filing of applications for product patents in the areas of drugs, pharmaceuticals
and agro chemicals though such patents were not allowed. However, such
applications were to be examined only after 31-12-2004. Meanwhile, the
applicants could be allowed Exclusive Marketing Rights (EMR) to sell or
distribute these products in India, subject to fulfillment of certain conditions.
• The second amendment to the 1970 Act was made through the Patents
(Amendment) Act, 2002 (Act 38 0f 2002). This Act came into force on 20th
May 2003 with the introduction of the new Patent Rules, 2003 by replacing the
earlier Patents Rules, 1972

74 

 
 

• The third amendment to the Patents Act 1970 was introduced through the
Patents (Amendment) Ordinance, 2004 w.e.f. 1st January, 2005. This Ordinance
was later replaced by the Patents (Amendment) Act 2005 (Act 15 of 2005) on 4
th April, 2005 which was brought into force from 1-1-2005.
Under the provisions of section 159 of the Patents Act, 1970 the Central
Government is empowered to make rules for implementing the Act and
regulating patent administration. Accordingly, the Patents Rules, 1972 were
notified and brought into force w.e.f. 20.4.1972. These Rules were amended
from time to time till 20 May 2003 when new Patents Rules, 2003 were brought
into force by replacing the 1972 rules. These rules were further amended by the
Patents (Amendment) Rules, 2005 and the Patents (Amendment) Rules, 2006.
The last amendments are made effective from 5 th May 2006.
Under US-The first Patent Act of the U.S. Congress was passed on April 10,
1790, titled "An Act to promote the progress of useful Arts”. The first patent
was granted on July 31, 1790 to Samuel Hopkins for a method of producing
potash (potassium carbonate). A revised patent law was passed in 1793, and in
1836 a major revision to the patent law was passed. The 1836 law instituted a
significantly more rigorous application process, including the establishment of
an examination system. Between 1790 and 1836 about ten thousand patents
were granted. By the Civil War about 80,000 patents had been granted.
Under British-case laws-
• Darcy v/s allein
• Cloth workers of Ipswich case

75 

 
 

Q.6. Explain the provision regarding revocation of patents?


Ans- Revocation of Patent' is cancellation of patent rendering a patentee devoid
of rights given to him for his patent. Patents can be revoked by:
• Intellectual Property Appellate Board (IPAB) considering a petition applied
by any person interested based on the grounds given under section 64(1), this
decision can be further challenged in High Court
• High Court, where counter-claim for legality of the patent is questioned in
the case of infringement,
• High Court, on appeal by the Central Government, being satisfied that the
applicant is not complying with the needs of the Central Government to use the
patented product or process under section 64(4),
• Controller, on the directions of Central Government, can revoke patents if
it is related to atomic energy or against public interest as given under section 65
and 66, respectively,
• Controller, considering application filed by any interested person or Central
Government u/s 85, may revoke a patent after two years of the grant of the first
compulsory license based on the non- fulfillment of the basic requirements like
workability in the territory of India, satisfaction of public requirements and
availability at affordable price.
Judgment in the case of F. Hoffmann-LA Roche Ltd. and Ors. Vs. Cipla Ltd
2009 Clarifies the fact that the mere grant of a patent does not guarantee its
resistance to subsequent challenges, which can still be faced in the form of a
counter claim in a suit or a petition applied on the grounds mentioned in Section
64. A patent, u/s 64(1) of the Patents Act 1970, can be revoked by High Court or

76 

 
 

IPAB on a number of matters within one year of grant of the patent9, briefly
stated as under:
• Subject do not constitute invention based on non-compliance of basic
requirements of novelty, no obviousness, usefulness and sufficient disclosure
• Any of the claims is also claimed in the complete specifications of another
patent with earlier priority date
• Person is not entitled to be an applicant of the application under the
provisions of the act
• Patent was obtained wrongfully from petitioner or other concerned person
• Subject covered in the application is publicly known or used or anticipated
from any work published before the priority or filing date of the application
• Subject matter is not patentable under the scope of the act
• Claim(s) are non-descriptive or doesn't disclose the best method of
performing the process
• Scope of claim(s) not clearly defined
• Secretly used in India before the priority date of the respective claim
• Failure of disclosure of information as required under section 8
• Non-compliance of the secrecy directions provided under section 35 and/or
applying for grant outside India without prior permission as directed under
section 39
• Non-disclosure or false intimation of geographical origin of the biological
material used in the application
• Knowledge involved in application is previously known to any local
community in India or elsewhere

77 

 
 

Revocation, as the term is used in the Patents Act 1970, is mainly a post-grant
operation. But apart from the reasons stated above and the provision stated
under section 65, 66 and 85, a patent is also said to be revoked under below
stated conditions10:
• Cancellation of patent application and rights of a granted patent following a
successful trial of pre-grant and post grant opposition, respectively
• If not contested for post-grant opposition within a period of two months
from the date of receipt of notice given by the Controller
• Patent shall also be revoked when the offer of surrender by applicant is
accepted by the Controller
Although the grounds mentioned for revocation under section 64(1) are quite
similar to the grounds on which pre or post-grant opposition is made (as
mentioned under section 25(1) and 25(2), respectively), the same cannot be
quoted for making a post grant opposition. This act of an opponent was also
criticized and corrected by the opposition board in the hearing (2017) of a
post-grant opposition filed by P. Sunil against G.R. Kaliaperumal having patent
application number 224/CHE/2010. P. Sunil choses section 64 to be the ground
for post-grant opposition, which although was not objected by the patentee but
corrected by the opposition board in the course of proceedings with the
corresponding clauses of section 25(2).
Many a times such sections are used in disguise by the petitioners to revoke the
patent especially to get monetary benefits or to end a linked contract. Monsanto
Technology LLC and Ors. vs. Nuziveedu Seeds Limited and Ors 2017is a
case11 where petition under various subsections of section 64 was filed by
Nuziveedu Seeds Limited against the Monsanto Technology LLC in order to get

78 

 
 

away with the contractual license and in turn the high trait fee asked by the
company. This petition was granted by the IPAB and the patent owned by
Monsanto Technology LLC was revoked. But later when the decision was
challenged in High Court, the decision was reversed stating that Nuziveedu
Seeds Limited and Ors. Did not seem to be interested in renewed arrangement
with Monsanto Technology LLC and thus took the position that the grant of
patent itself is bad in law and thus, asserting their right to continued use of the
technology with liberty. It was ordered that the contract will be on toll as before
between both the parties but with the revised rates of trait fee as decided by the
Government of India.

Q.8. explain infringement of patent rights and the remedies available in


case of infringement?
Ans- Infringement and Defences-
When a person works a patentable invention without a law full authority to use
it, he is said to have committed infringement of the patent right. The person in
whom this right is vested, be it the owner of patent or an assignee or a licensee
or anybody else who lawfully has an interest in the patented invention, can bring
the proceeding of infringement against the unauthorized user. The defences
available are that the invention is not really an invention under the meaning of
the Act, that it had been obtained wrongfully in contravention of rights of rights
of somebody else who really claimed it, that it was obvious and that it had been
subject- matter of an earlier patent etc. Section 107 states that all the ground on
which a paten may be grounds of defense in a suit of infringement. The Court
has also upheld this view in The Pilot pen Company India Private Ltd. V. The

79 

 
 

Gujarat Industries Private Ltd. Bombay (A.I.R. 1967 Madras 215) The question
of infringement of a patent is a mixed question of law and fact. Observation thus
the Curt, In Lallubhai Chakubhai Jariwala v. Chimanlal Chunnilal and Company
(A.I.R. 1936 Bombay 99.) stated that a patent may be infringed in several ways,
one of which being the use of “the invention or by any colorable imitation
thereof in the manufacture of articles or by getting the invention in practice in
any other way.” A patent, according to Courts view, may sometimes be
infringed by taking a part only of the invention, but that depends on whether the
part only of the invention. But that depends on whether the part for which
protection is asked is a new and material part.
Remedies
1. Civil Remedies
As far as civil is concerned, an exclusive licensee, a Patentees or a holder of a
compulsory license can bring a suit of infringement in a court not below the
District Court. However, where counter claim. Is made by the alleged infringer
for revocation of a patent, the suit along with the counter-claim shall be
transferred to High Court. The burden of proof always lies with the defendant to
prove that the process used by him is different from the patented process. Where
a person, whether entitled to interested in a patent or not, threatens ant other
person of infringement proceedings, the latter can bring a suit praying for:
a. A declaration by the Court that these threats are unjustifiable,
b. An injunction against the threats; and
c. Damages if any.
Section 107-Adeclares that any act of making a patented product for uses relate
to development and submission of information, whether in India or abroad; and

80 

 
 

important ion of a patented product from a person duly authorized under law to
produce, sell or distribute the said product shall not be taken to be
infringements.
2. Administrative Remedy
The Appellate Board establish under sec 83 of Trade Mark Act, 1999 shall be
the Appellate Board for the purposes of this Act also, except that the technical
member of this board shall have the following qualifications,
a. He has been or has acted as Controller under this Act for at least five years;
b. He has at least ten years’ experience as Registered Patent Agent and
possesses a degree in Engineering or technology from a recognized University.
The Central Government is empowered by the Act to determine the nature and
categories of officers and others and other employees to assist the Appellate
Board in discharge of its functions. The Appellate Board cannot hear appeals
against any decision, order or direction issued under this Act by Central
Government, or from any act or order of the Controller for the purpose of giving
effect to any such decision, order or direction.
3. Criminal Remedies
The Act provides punishments for violation of its provision. If fails to comply
with any direction given under Section 35(Secret Patents) or causes to be made
an application for grant to patent in violation of section 39, he shall be
punishable with a maximum term of two years or fine or both. The punishment
is the same in case of falsification of entries in the register of patents. An
unauthorized claim of patent claim of patent rights is punishable with fine up to
one lakh rupees; and a wrongful use of the word “patent office” is punishable
with an imprisonment of up to six months or fine or both.

81 

 
 

UNIT –IV
Q.1 Describe the history relating to copyright in India?
Ans-Today almost every nation has a copyright law in place and is mostly
standardized to some extent through international and regional agreements such
as the Berne Convention and the European copyright directives. But when we
look back, we realize that the Copyright law has a very unique history.
The earliest copyright case is traced back to Ireland, where there was a dispute
over the ownership of the Irish manuscript Catha. The Catha is the oldest extant
Irish manuscript of the Psalter. It contains a Vulgate version of Psalms XXX
(10) to CV (13) with an interpretative rubric or heading before each psalm. It is
traditionally ascribed to Saint Columba as the copy, made at night in haste by a
miraculous light, of a Psalter lent to Columba by St. Finnian. A dispute arose
about the ownership of the copy and King Diarmait Mac Cerbhaill gave the
judgement “To every cow belongs her calf; therefore to every book belongs its
copy. The real need of copyright law was felt only after the invention of printers
and copiers. Prior to the invention of printers, writing could be created only
once. It was highly laborious and risk of errors was involved in the manual
process of copying by a scribe. Interestingly even in such a situation, Europe
had an elaborate system censorship and control over scribes.
During the 15th and 16th Century, printing was invented and widely established
in Europe. Copies of Bibles were the first to be printed. Government had
allowed printing without any restrictions but this led to dissemination of lot of
governmental information. Subsequently, government started issuing licenses
for printing.

82 

 
 

The republic of Venice was the first to grant privilege to print books. It was for
the history of its own named ‘Rerum venetarum ab urbe condita opus’ authored
by Marcus Antonius Coccius Sabellicus. From 1492 onwards Venice began to
regularly grant privileges for books.
In 1518, the first copyright privilege was granted in England. It was issued to
Richard Pynson, King’s Printer, the successor to William Caxton. The privilege
gave a monopoly for the term of two years. These copyright privileges were
called as monopolies. Later in 1701, the parliaments of England and Scotland
were united as a result of the Anglo-Scottish Union. The new parliament was
able to change the laws in both countries and an important early piece of
legislation was the Copyright Act of 1709, also known as the Statute of Anne,
after Queen Anne. The act came into force in 1710 and was the first copyright
statute.
United States did not follow the law of England. Till 1783, US did not have
proper copyright legislations. It was in 1783, few writers raised voice against the
government to enact copyright law as they believed that nothing is more
properly a man’s own than the fruit of his study, and that the protection and
security of literary property would greatly tend to encourage genius and to
promote useful discoveries.” But then, the continental congress did not have any
power to enact copyright laws. In 1787, proposals were submitted at the
Philipedia convention to grant congress the powers to enact copyright law, and it
was finally in1790, first federal Copyright Act was enacted.
Thereafter, came the international agreements like the Paris convention and the
Berne convention which are the standards for the copyright law enacted by
every country including India.

83 

 
 

It is believed that the evolution of Copyright law in India has been in three
phases. The law of copyright was introduced in India only when the British East
India Company was established in 1847. This Act had very different provisions
in comparison to today’s law. The term of the Copyright was life time of the
author plus seven years after the death of the author. But in no case could the
total term of copyright exceed a period of forty-two years. The government
could grant a compulsory license to publish a book if the owner of copyright,
upon the death of the author, refused to allow its publication. Registration of
Copyright with the Home Office was mandatory for enforcement of rights under
the Act. This was the first phase.
The second phase was in 1914, when the Indian legislature under the British
Raj enacted the Copyright Act of 1914. It was almost similar to the United
Kingdom Copyright Act of 1911. But the major change that was brought in this
Act was criminal sanction for infringement. Number of times amendment were
brought to this Act up till 1957. Subsequently, India saw the third phase of
Copyright law in 1957. The Copyright Act, 1957 was enacted in order to suit the
provisions of the Berne Convention. This Act was enacted by Independent India
by which we are governed till date.

Q.2.What is the requirements for registration of a copyright under the


copyright act?
Ans-Requirements and Procedure for Copyright-
Requirements-
1. Name, address and nationality of the applicant

84 

 
 

2. Name, address and nationality of the author of the work


3. Nature of applicant’s interest in the copyright i.e. OWNER / LICENSEE
etc.
4. Title of the work
5. A declaration signed by the author (if different from the applicant)
6. Language of the work
7. Whether the work is published or unpublished
8. If the work is published, year and country of first publication and name,
address and nationality of the publisher
9. Year and countries of subsequent publications, if any
10. Name, address and nationality of any other person authorized to assign or
license the rights in the copyright
11. Power of attorney for the firm
12. Six hard copies of the work and three soft copies
13. (For computer programs – 3 copies of the program on CD ROMs.)

Copyright Registration Procedure-


The procedure for registration is as follows:
1. Application for registration is to be made on as prescribed in the first
schedule to the Rules;
2. Separate applications should be made for registration of each work;
3. Each application should be accompanied by the requisite fee prescribed in
the second schedule to the Rules; and
4. The applications should be signed by the applicant or the advocate in
whose favor a Vakalatnama or Power of Attorney has been executed. The Power

85 

 
 

of Attorney signed by the party and accepted by the advocate should also be
enclosed.
Time for Processing Application
After you file your application and receive diary number you have to wait for a
mandatory period of 30 days so that no objection is filed in the Copyright office
against your claim that particular work is created by you.
Scope and Extent of Copyright Registration
Both published and unpublished works can be registered. Copyright in works
published before 21st January, 1958, i.e., before the Copyright Act, 1957 came
in force, can also be registered, provided the works still enjoy copyright. Three
copies of published work may be sent along with the application.
If the work to be registered is unpublished, a copy of the manuscript has to be
sent along with the application for affixing the stamp of the Copyright Office in
proof of the work having been registered. In case two copies of the manuscript
are sent, one copy of the same duly stamped will be returned, while the other
will be retained, as far as possible, in the Copyright Office for record and will be
kept confidential. It would also be open to the applicant to send only extracts
from the unpublished work instead of the whole manuscript and ask for the
return of the extracts after being stamped with the seal of the Copyright Office.
When a work has been registered as unpublished and subsequently it is
published, the applicant may apply for changes in particulars entered in the
Register of Copyright in Form V with prescribed fee.
All kinds of literary and artistic works can be copyrighted; you can also file a
copyright application for your website or other computer program. Computer
Software or programmer can be registered as a ‘literary work’. As per Section 2

86 

 
 

(o) of the Copyright Act, 1957 “literary work” includes computer programmes,
tables and compilations, including computer databases. ‘Source Code’ has also
to be supplied along with the application for registration of copyright for
software products. Copyright protection prevents undue proliferation of private
products or works, and ensures the individual owner retains significant rights
over his creation.

Q. 3 what are the remedies available in case of infringement of copyright?


Ans- Infringement-
In dealing with copyright, we should bear in the mind that copyright does not
protect novelty but only originality. Copyright protects only the expression and
not the idea. Therefore, if it is the only method of expressing the work, it cannot
be protected. Best example would be the Telephone Directory wherein the
Name, Address, Phone No. are given and also given in alphabetical order. There
can be no other way of expressing the same. Therefore, this would not amount to
copyright infringement. This is popularly referred to as Idea-Expression
Dichotomy. Once the rights of the owner have been established, the next step is
to prove that that there is an actual infringement. If the defendant makes copies
of a copyrighted work and commercially exploits such copies or any blatant
infringement, nothing further needs to be proved to establish infringement apart
from what has been discussed above.
However, more complicated questions arise when the defendant the alleged
infringing work involved relates to something, which is similar, but not identical
with the plaintiff’s work. In such cases, in order to prove infringement, the
plaintiff must show the following:

87 

 
 

a. The defendant copied directly from the plaintiff’s work, and


b. The elements copied, when taken together, amounts to an improper
appropriation.
Realizing that direct evidence of copying will be rarely available; courts have
universally allowed copyright owners to prove copying on the basis of
circumstantial evidence, specifically through inferences from the defendant’s
access to the plaintiff’s work and from any similarities between two works.
In the case of Super Cassette Industries vs. Nodules Co. Ltd. the defendant played
cassette in Hotel amounts to copyright infringement. This was clearly held to be
act of infringement of author’s right over copyright. Copying can, therefore, be
proved by inference. It can be inferred that the defendant has in fact copied the
plaintiff’s work from the fact that the defendant had access to the plaintiff’s work
and from the similarities between his work and that of the plaintiff’s. The
rationale behind this is that given the sufficient opportunity that the defendant had
to copy the plaintiff’s work in addition to the striking similarity between the two
works, the evidence in hand is indicative of copyright infringement.
In the case of Roma Metra vs. State of Bihar, the Plaintiff, a student gave the
work to the guide. The guide published the work as her own. The published
article was substantially similar and therefore, amounted to copyright
infringement. In the case of Ty Ink vs. GMA Accessories, it was held that
Similarity between works is highly unlikely to have been in accident of
independent creation. This is an evidence of access. Therefore, there is a
reciprocal relationship between proof of access and similarity and this
relationship is subject to two important limitations In the case of S.K. Dutt vs

88 

 
 

Law Book Co. and others, the court determined the amount of substantiality
should be more than half of the total work. It has also held that where the half of
the work is copied and the remaining being original work, it does not constitute
infringement.
a. “If there are no similarities, no amount of evidence of access will suffice to
prove copying”.
b. “If evidence of access is absent, the similarities must be so striking as to
preclude the possibility that plaintiff and the defendant arrived at the same result.”
Therefore, to summaries the Condition to prove infringement can be summarized
as follows:
a. Closely Similarity
b. Unlawful
c. Some connection
d. Access to original work
Fair Use
For the purpose of deciding fair use of the work, the following factor has to be
taken into consideration before determining it to be and copyright Infringement.
a. the purpose and character of the use, including whether such use is of a
commercial nature or is for nonprofit educational purposes;
b. the nature of the copyrighted work;
c. the amount and substantiality of the portion used in relation to the copyrighted
work as a whole; and
d. the effect of the use upon the potential market for or value of the copyrighted
work.

89 

 
 

At the outset, it should be mentioned that the “Fair Use” of the work depends
upon facts and circumstances of each case. In a copyright infringement case
dealing with fair use, the duty of the court is to first determine whether the
defendant has use the copyrighted information in a natural or justifiable manner
or has taken advantage of already existing work of the plaintiff. In deciding that,
the court has to deal with the above mentioned factors before coming to any
conclusion.
For example,
If a person writes a book on topic “Mother’s care” and gets his book
copyrighted and subsequently, another person with title “Mother’s care” writes a
article on mother’s care on child. Under ordinary circumstances, it may be an
infringement, but here it is just an article on care of mother towards the child to
a non-paying audience with no commercial element involved in it. Therefore, it
would be come under fair use not amounting to infringement.

Piracy in Internet
Internet piracy is the unlawful reproduction and/or distribution of any
copyrighted digital file that change hands over the Internet. This can be done
with music files, videos and movies, e-books, software, and other materials.
Those who engage in this type of piracy can often conduct their entire operation
on the Internet, including advertising and sales. It has become a worldwide
crime problem, because of the relative ease with which it can be committed,
even over long distances.
Piracy is the breach of copyright that is perpetrated by illegal reproduction of,
most commonly, music and video. The major technological shifts are easily

90 

 
 

ascertainable as they tend to coincide with the enactment of legislative


amendments to tackle new mediums and the inauguration of international
institutions to protect and enforce the cause. An example of such an institution is
the International Federation of the Phonographic Industry (IFPI), which was set
up in 1993, and 1971, as a major turning point in video technology, is the year
of amendment of the Berne Convention and the Geneva Phonograms
Convention.

1 CIVIL REMEDIE
The most importance civil remedy is the grant of interlocutory injunction since
most actions start with an application for some interlocutory relief and in most
cases the matter never goes beyond the interlocutory stage. The other civil
remedies include damages - actual and conversion; attorney’s fees, rendition of
accounts of profits and delivery up.

a. INTERLOCUTORY INJUCTIONS

The principles on which interlocutory injunctions should be granted were


discussed in detail in the English case of American Cyanamid v Ethicon Ltd31.
[1975] AC 368 (HL(E)]. After this case, it was believed that the classic
requirements for the grant of interim injunction are:

a. Prima facie case


b. Balance of Convenience; and
c. Irreparable injury

91 

 
 

In the case of Series 5 Software Ltd. v Philip Clarke & Others, Laddie J
re-examined the principles and took a fresh look at what Cyanamid had actually
decided. The learned judge held:

a. The grant of an interlocutory injunction was a matter of discretion and


depended on all
b. the facts of the case; there were no fixed rules;
c. the court should rarely attempt to resolve complex issues of disputed fact or
law;
Major factors the court should bear in mind were
(i) the extent to which damages were likely to be an adequate remedy and the
ability of the other party to pay
(ii) The balance of convenience
(iii) The maintenance of the status quo, and
(iv) Any clear view the court may reach as to the relative strength of the parties'
case. Thus, this case places emphasis on the merits and the effect may well be to
obtain a nonbinding view by a judge on the merits. This may lengthen the hearing
of application for interlocutory injunction as parties may lead evidence on the
merits but it may have the overall effect of putting an early end to the main
action.
b. PECUNIARY REMEDIES
Under the Copyright laws of some countries like the United Kingdom, it is
essential for the plaintiff to elect between damages and an account of profits
although in the two recent cases, namely Baldock v Addison [1994] FSR 665
and Island Records v Tring International Plc [1995] FSR 560, the Court held

92 

 
 

that there could be a split trial and a procedure could be adopted by which the
trial could be divided so that once liability has been established, thereafter the
plaintiff would be able to seek discovery in order for him to make an informed
decision on which of the two of the remedies to elect, namely damages or
account of profits. In Cala Homes (South) Ltd. v Alfred Calpine Homes East Ltd
[I995] FSR 818, Ladies J held that additional statutory damages could be
granted even where the plaintiff elected for account of profits. Under Sections
55 and 58 of the Indian Copyright Act, 1957, the plaintiff can seek the following
three remedies, namely
a. account of profits
b. compensatory damages and
c. conversion damages which are assessed on the basis of value of the article
converted.

c. ANTON PILLER ORDER


The Anton Pillar Order derives its name from a Court of Appeal decision in
Anton Pillar AG vs. Manufacturing Processes. An Anton Pillar Order has the
following elements:
5. An injunction restraining the defendant from dealing in the infringing goods or
6. Destroying, them; an order that the plaintiffs solicitors be permitted to enter the
premises of the
7. Defendants, search the same and take goods in their safe custody; and an order
that defendant be directed to disclose the names and addresses of suppliers
8. Customers and also to file an affidavit will a specified time giving this
information.

93 

 
 

d. MAREVA INJUNCTION
Mareva Injunction is an order which temporarily freezes assets of a defendant
thus preventing the defendant from frustrating the judgment by disposal of such
assets.

1. CRIMINAL REMEDIES
Criminal remedy includes imprisonment of the infringer and the infringing
copies seized. Besides one can get ANTON PILLER order from court, which
means that court grants an ex-parte order if it feels that the case is balanced in
favor of copyright holder. The owner can claim damages from the infringer. The
author can get an order for search of defendant’s premises, if there is clear
evidence to show the presence of infringing copies in the premises of infringer.
The infringer is being liable for imprisonment ranging 6 months to 3 years
and/or fine of Rs.50000/- to Rs. 2 lacks. For the first time, the punishment would
be for a period of 6 months to 3 years and/or Rs.25000 to 2 lacks and for the
second Time it would be for 1 to3 years and/or Rs.50000 to 2 lacks. And for
infringement on computer Program, the punishment may vary from 7 days up to
3 years and/or Rs.50000 – 2 lacks.

2. ADMINISTRATIVE REMEDIES
An application can be made by the owner of copyright in any work or by his
duly authorized agent, to the Registrar of Copyrights to ban the import of
infringing copies into India and the delivery of infringing copies of copyrighted
article which were earlier confiscated from infringer to the owner of the
copyright.

94 

 
 

Short type Questions-


Q.6. Briefly discusses the copyright in literary artistic and musical work?
Ans-Literary artistic, musical works-
Literary work- Copyright subsists in original literary works and relates to the
expression of thought, but the expression need not be original or novel.
• The work must not be copied from another work but must originate from
the author.
• Two authors independently producing an identical work will be entitled for
copyright in their respective works.
• The emphasis is more on the labor, skill judgment and capital expended in
producing the work. It includes tables, compilations and computer programs.

Musical work- Copyright subsists in original musical work and


• Includes any combination of melody and harmony, either of them reduced
to writing or otherwise graphically produced or reproduced.
• An original adaptation of a musical work is also entitled to copyright.
• There is no copyright in a song. A song has its words written by one man
and its music by another; is words have a literary copyright, and so has its
music. These two copyrights are entirely different and cannot be merged.
• In cases where the word and music are written by the same person, or
where they are owned by the same person, he would own the copyright in the
song.
Artistic work means –
• A painting,

95 

 
 

• A sculpture,
• A drawing including a diagram, map, chart or plan,
• An engraving or a photograph, whether or not any such work possesses
artistic quality;
• An architectural work of art; and any other work of artistic craftsmanship.
• The work need not possess any artistic quality but he author must have
bestowed skill, judgment and effort upon the work.
• A poster used in advertisement is an artistic work. But advertisement
slogans consisting of a few words only are not copyright matter.

Artistic Work
According to Section 2(c), artistic work means:-
a. painting, a sculpture a drawing including a diagram, map, chart, or plan,
and engraving on a photograph whether or not such work possesses artistic
quality;
b. a work of architecture; and
c. any other work of architecture craftsmanship.

Painting
Painting is an artistic work even if it possesses no aesthetic quality. What is
sufficient to entitle it for a copyright is that it must be original, i.e., the painting
should be the creation of the painter and not a mere copy of another painting. A
painting in order to be copyrighted must be on a surface; it is a tangible object
and not only an idea. Only a painting on a tangible surface is entitled to
copyright protection.

96 

 
 

Photograph
A photograph is an artistic work entitled to copyright. A photograph must be
original, i.e., originally taken by the photographer to be entitled to protection
and to be original, investment of some degree of skill and labor must be evident.
A photograph of a public object, e.g., a public building when photographed in a
particular manner by a particular individual is a subject matter of copyright.
This does not prohibit another person from taking a photograph of the same
object from same angle and claim an independent copyright. A photograph or a
Xerox copy of a photograph itself cannot be a subject matter of copyright since
there is no involvement of original skill and labor in making such a copy.
In Associated Publisher v. Bash yam, where a portrait of Mahatma Gandhi was
made based on two photographs, it was held that a portrait based on photographs
will be entitled to copyright if it produced a result different from the photograph
and the portrait itself is original.

Musical Work
Section 2(p) defines 'Musical Work' as a work consisting of music and includes
any graphical rotation of such work but does not include any words or any
action, intended to be sung, spoken or performed with music.

Song
There is no copyright in a song, the words of the song create a copyright in the
author of the song and the music of the song is the copyright of the composer

97 

 
 

but the song itself has no copyright. In a case where a song is written and the
music composed by the same man, he would own the copyright in the song.

Old songs with different music compositions (remix)


The remix songs which are very popular among the young generation is within
the definition of adaptation of a musical work, such adaptation is not an
infringement of the copyright of the original musical composition.
Supreme Court in a significant judgment has held that version recording if done
by a skillful and laborious rearrangement of different music with due permission
of the original owner of songs, may claim protection.

Cinematograph Film
Section 13(1) (b) incorporates cinematograph films as eligible for copyright
protection. Cinematograph is a film which by rapid projection through an
apparatus called projector produces the illusion of motion on a screen of many
photographs taken successfully on a long film.
The sound track associated with the film is also a part of the cinematograph
film. In Balwinder Singh v. Delhi Administration, and Tulsidas v. Vasantha
Kumar, video films and television have been treated by cinematograph film
within the meaning of Sec. 13(l)(b). As such they are also eligible for copyright
protection.

Sound Recording
In Section 13(1) (c) (as added by Amendment Act of 1994) the sound recording
has also been made eligible for copyright protection. According to Section 2

98 

 
 

(xx), sound recording means recording of sound from which such sound may be
produced regardless of medium on which such recording is made or the method
by which the sound is produced.
The right of sound recording is different from the subject matter recorded as
they are subjects of independent copyrights. The author of the sound recording
is the producer. Musical works and sound recordings embodying the music are
considered separate subject-matter for copyright. The copyright in the recording
of music is separate from the copyright in the music. Copyright in the music
vests in the composer and the copyright in the music recorded vests in the
producer of the sound recording. Where the song has not been written down and
the composer who is also the performer records the song two copyrights come
into existence simultaneously, one for the music and one for the sound
recording.

Q.7.define copyright. What are the main features of a copyright?


Ans- Meaning of copyright.—sec 14-For the purposes of this Act, “copyright”
means the exclusive right subject to the provisions of this Act, to do or authorize
the doing of any of the following acts in respect of a work or any substantial part
thereof, namely:—
Copyright under The copyright act 1957:
Literary, dramatic, musical and artistic works
Dramatic work- Copyright subsists in original dramatic work and its adaptation.
Musical work- Copyright subsists in original musical work and
Artistic work

99 

 
 

Main features of Copyright Act of 1957-


1. Creation of statute
The copyright is granted and protected according to Copyright Act. There is no
common law right.1 No copyright can exist in any work except as provided in
Section 16 which reads as under ;-

2. "No copyright except as provided in this Act No person shall be entitled to


copyright or any similar right in any work, whether published or unpublished,
otherwise than under and in accordance with the provisions of this Act or any
other law for the time being in force, but nothing in this section shall be
construed as abrogating any right or jurisdiction to restrain breach of trust or
confidence."

3. Multiple Rights

Copyright is not a single right; rather it is a bundle of rights in the same work. It
comprises of, in case of literary work, right to reproduction, right to translation,
right to adaptation, right to dramatic and cinematographic version, right to
public per formation and right to serial publication. It is also in the form of
monopoly right restraining others from exercising all or any of the rights and it
is also a negative right, restraining others from copying or reproducing the work.
A detailed study of these rights has been made in the next chapter.

4. The Work must be original

100 

 
 

The first and essential requirement for claiming copyright is the originality of
work. There cannot be a claim of copyright in a copy of any other's work or in a
similar work Use of original skill and labour is essential to acquire copyright in
a works .

Copyright exists in expression of idea and not mere idea-


The copyright exists only in expression of idea in some material form, i.e. book,
film, photograph or musical tone. Mere idea unless expressed is not capable of
claiming any copyright. For example Ii person may have a brilliant idea or plot
for a story or a picture in his mind, he then narrates his idea to another person,
who works on the idea and writes a book or play or paints a picture the person
who communicated the idea cannot claim the copyright, and it is the second
person who gave expression to the idea and be will become entitled to claim
copyright. The Supreme Court of India in a leading case R.G. Anand v. Deluxe
Films has declared that there can be no copyright in an idea, subject matter,
themes, plot or historical facts.

Subject-matter of Copyright Protection


Section 13(1) of the Act has provided categories of works, in which copyright
subsists. It reads as follows:
"Subject to the provisions of• this section and the other provisions of this Act,
copyright shall subsist throughout India in the following classes of works, that is
to say-

a. Original literary, dramatic, musical and artistic works;

101 

 
 

b. Cinematograph films; and


c. Sound recordings."

Very short type questions-


Q.8. Define copyright?
Ans- Meaning of copyright.—sec 14-For the purposes of this Act, “copyright”
means the exclusive right subject to the provisions of this Act, to do or authorize
the doing of any of the following acts in respect of a work or any substantial part
thereof, namely:—
Copyright under The copyright act 1957:
Literary, dramatic, musical and artistic works
Literary work- Copyright subsists in original literary works and relates to the
expression of thought, but the expression need not be original or novel.
• The work must not be copied from another work but must originate from
the author.
• Two authors independently producing an identical work will be entitled for
copyright in their respective works.
• The emphasis is more on the labor, skill judgment and capital expended in
producing the work. It includes tables, compilations and computer programs.

Dramatic work- Copyright subsists in original dramatic work and its adaptation.
• It includes any piece or recitation, choreographic work
• Entertainment in dumb show
• The scenic arrangement or acting form of which is fixed in writing
otherwise

102 

 
 

• But does not include a cinematograph film.


Musical work- Copyright subsists in original musical work and
• Includes any combination of melody and harmony, either of them reduced
to writing or otherwise graphically produced or reproduced.
• An original adaptation of a musical work is also entitled to copyright.
Artistic work means –
• A painting,
• A sculpture,
• A drawing including a diagram, map, chart or plan,
• An engraving or a photograph, whether or not any such work possesses artistic quality;
• An architectural work of art; and any other work of artistic craftsmanship.
• The work need not possess any artistic quality but he author must have bestowed
skill, judgment and effort upon the work.
• A poster used in advertisement is an artistic work. But advertisement slogans consisting
of a few words only are not copyright matter.

Q.9. Can copyright be relinquished?

Ans- Right of author to relinquish copyright-sec 21-

(1) The author of a work may relinquish all or any of the rights comprised in the copyright in the
work by giving notice in the prescribed form to the Registrar of Copyrights and thereupon such
rights shall, subject to the provision of sub section (3), cease to exist from the date of the notice.
(2) On receipt of a notice under sub section (1), the Registrar of Copyrights shall cause it to be
published in the Official Gazette and in such other manner, as he may deem fit.
(3) The relinquishment of all or any of the rights comprised in the copyright in a work shall not
affect any rights subsisting in favor of any person on the date of the notice referred to in sub
section. (1).

103 

 
 

Q.10. what is the term of copyright in an anonymous work?

Ans- Term of copyright in anonymous and pseudonymous works –sec 23-

In the case of a literary, dramatic, musical or artistic work (other than a


photograph), which is published anonymously or pseudonymously, copyright shall
subsist until [sixty] years from the beginning of the calendar year next following
the year in which the work is first published:.
Provided that where the identity of the author is disclosed before the expiry of the
said period, copyright shall subsist untill sixty years] from the beginning of the
calendar year next following the year in which the author dies.
(2) In sub section (1), references to the author shall, in the case of an anonymous
work of joint authorship, be construed.-
a. where the identity of one of the author is disclosed, as references to that
author,
b. where the identity of more author than one is disclosed, as reference to the
author who dies last from amongst such authors.
(3) In such section (1), references to the author shall, in the case of a
pseudonymous work of joint authorship, be construed,-
a. where the names of one or more (but not all) of the authors are
pseudonymous and his or their identity is not disclosed, as references to the
author whose name is not a pseudonym, or, if the names of two or more the
authors are not pseudonymous, as references to such of those authors who
dies last,
b. where the names of one or more ( but not all) of the authors are
pseudonyms and the identity of one or more of them is disclosed, as
references to the author who dies last from amongst the authors whose
names are not pseudonymous and the authors whose names are pseudonyms
and disclosed, and
c. where the names of all the authors are pseudonyms and the identity of one
of them is disclosed, as references to the authors whose identity is disclosed
or if the identity of two or more of such authors is disclosed, as references to
such of those authors who dies last.
104 

 
 

UNIT-5

Long type questions-


Q.1.what do you mean by the infringement of a trademark? Discuss about remedies?
Ans- INFRINGEMENT OF REGISTERED TRADE MARKS -
Infringement, very broadly means taking unfair advantage or being detrimental to the distinctive
character or reputation of a trade mark. Under the Trade Marks Act, 1999 the meaning of
infringement has been enlarged as more actions shall be taken as constituting infringement which
are listed in Section 29. Section 29 dealing with infringement of trademarks explicitly
enumerates the grounds which constitute infringement of a registered trade mark. This section
lay down that when a registered trade mark is used by a person who is not entitled to use such a
trade mark under the law, it constitutes infringement. This section clearly states that a registered
trade mark is infringed, if the mark is identical and is used in respect of similar goods or
services; or the mark is deceptively similar to the registered trade mark and there is an identity or
similarity of the goods or services covered by the trade mark; or the trade mark is identical and is
used in relation to identical goods or services; and that such use is likely to cause confusion on
the part of the public or is likely to be taken to have an association with the registered trade
mark. Sub-section (4) states that a person shall be deemed to have infringed a registered trade
mark, if he uses a mark which is identical with or similar to the trade mark, and is used in
relation to goods or services which are not similar to those for which trade mark is registered;
and the registered trade mark has a reputation in India and the use of the mark without due cause
would take unfair advantage of or is detrimental to the distinctive character or repute of the
registered trade mark.
Sub-section (5) prohibits a person from using someone else’s trade mark, as his trade name or
name of his business concern or part of the name of his business concern dealing with goods or
services in respect of which trade mark is registered. A person shall be deemed to have used a
registered trade mark in circumstances which include affixing the mark to goods or packaging,
offering or exposing the goods for sale or supply of services, importing or exporting the goods,
using the trade mark as trade name or trade mark on business paper or in advertising.

105 

 
 

A person shall also be deemed to have infringed a registered trade mark if he applies such
registered trade mark to a material intended to be used for labeling or packaging goods as a
business paper, or for advertising goods or services knowing that the application of such mark is
not authorized by the proprietor or licensee. Advertising of a trade mark to take unfair advantage
of, or against the honest industrial or commercial practices or which is detrimental to the
distinctive character or is against the reputation of the trade mark shall constitute an infringement
under Section 29(8) of the Act. Where the distinctive element of a registered trade mark consists
of words, the spoken use of such words as well as visual representation for promoting the sale of
goods or promotion of service would constitute infringement under Section 29(9) of the Act. The
infringement action is a statutory remedy available to the registered proprietor or to the
registered user, based on statutory rights conferred by registration of a trade mark, subject to
other restrictions laid down in Sections 30, 34, 35 of the 1999 Act. An infringement action is
available to the registered proprietor or registered user to enforce his exclusive right over the
trade mark in relation to the goods in respect of which it is registered. If at the time of
registration of trade mark, certain limitations or conditions have been imposed, then, the
exclusive right has to be evaluated within the terms of such registration.
If an offending use of the mark fulfils the conditions laid down in Section 29(1) discussed above,
it squarely constitutes infringement In M/s J K Oil Mills v. M/s Adani Wilmar Ltd., 2010 (42)
PTC 639 (Del.), the Delhi High Court held that in order to constitute infringement under the
provisions of Section 29 of the Trade Marks Act, it would be necessary to show that impugned
trade mark (label) is identical or deceptively similar to the registered trade mark. Remedies
Whenever, a registered trade mark is infringed, in order to protect it the following remedies can
be resorted to:-
• Criminal remedies
• Civil remedies
• Administrative remedies

106 

 
 

For the purpose of infringement of an unregistered trade mark, the Common Law remedy of
Passing –off action can be invoked.
• Under Section 135 of the Trademarks Act, 1999, the court grant relief in any suit for
Infringement or for Passing Off includes Permanent & Interim injunction and either damages or
an account of profits together with or without any order for delivery up of the infringing labels
and marks for destruction or erasure. The order of interim injunction may be passed ex parte or
after notice. The Interim reliefs in the suit may also include order for:
• Appointment of a local commissioner, which is akin to an “Anton Pillar Order”, for search,
seizure and preservation of infringing goods, account books and preparation of inventory, etc.
• Restraining the infringer from disposing of or dealing with the assets in a manner which may
adversely affect plaintiff’s ability to recover damages, costs or other pecuniary remedies which
may be finally awarded to the plaintiff.

Criminal Remedies
Sections 101 to 121 deal with the matters relating to offences, penalties and procedure. Some of
the important provisions are discussed below.

1. Section 103 deals with the penalty for applying false trade mark, trade description, etc. and
imposes punishment with imprisonment for a term which shall not be less than six months but
which may extend to three years and with fine which shall not be less than fifty thousand rupees
but which may extend to two lacks rupees.
2. Section 105 prescribes enhanced penalty on second and subsequent conviction for offences
committed under sections 103 and 104 and imposes punishment with imprisonment which shall
not be less than one year but which may extend to three years and with fine which shall not be
less than one lack rupees but which may extend to two lacks rupees.
3. Section 106 provides penalty for removing piece goods, etc., in violation of the provisions of
Section 81 dealing with stamping of piece goods, cotton yarn and thread. This section provides
for forfeiture of goods to the government and fine up to ` 1000.

107 

 
 

4. Section 107 makes it an offence if a person falsely represents a trade mark as registered. It has
been clarified that use of symbols like “R” in circle in relation to unregistered trade mark would
constitute an offence. The punishment for such offences is imprisonment for a term which may
extend to three years or with fine or with both. It is also clarified that where the mark in question
is registered under the law of the country outside India, the use of the word or other expression to
denote such registration in foreign country is permissible.
5.According to section 108, the use of any words which would lead to the belief that a person’s
place of business is officially connected with the Trade Mark Office shall be treated as offence
and be punishable with imprisonment for a term which may extend to two years or with fine or
with both.
Section 109 contains provisions for penalty for falsification of entries in the register. This
offence is punishable with imprisonment not exceeding two years or with fine or with both.
Section 114 deals with offences by companies and provides that where a person committing
offence is a company, every person in charge of and responsible to the company for the conduct
of its business at the time of commission of an offence will be liable. Where a person accused
proves that the offence was committed without his knowledge or he has exercised all due
diligence to prevent the commission of such offence, he will not be liable. However, where it is
proved that an offence has been committed with the consent or connivance or is attributable to
any neglect of any Director, Manager, Secretary or any other officer of the company; he shall be
deemed to be guilty of the offence. Explanation to this section defines a company as to mean
body corporate and includes a firm or other association of individuals. The explanation also
defines director in relation to a firm, as to mean a partner in the firm.

Administrative Remedies;
A part from the civil remedies aforementioned, the Act also Best certain powers in the various
administrative authorized to grant reliefs to grant reliefs and remedies to aggrieved persons.
These powers may be exercised in respect of-
1. Classification of goods and services for the purpose of registration.
2. Publication of alphabetical index of classification of goods\services.

108 

 
 

3. Granting of or refusing to register a trade mark


4. Correcting and amending the register.
5. Renewal, removal and restoration of registration.
6. Assign ability and transmissibility of registered trademarks.
7. Registration of assignments and transmissions and
Other aspects relating to trade marks. It may be noted that it is the Registrar, who mostly
exercises these power under the guidance of the Central Government.

Q.2. what are different kinds of marks provided under the trademark act 1999? distinguish
between trademark and property mark?
Ans-A mark as per Section 2(m) of the Indian Trademarks law, includes a device, brand,
heading, label, ticket, name, signature, word, letter, numeral, shape of goods, packaging or
combination of colors or any combination thereof.
A trademark as per Section 2(zb) means “a mark capable of being represented graphically and
which is capable of distinguishing the goods or services of one person from those of others and
may include shape of goods, their packaging and combination of colours:”So a mark, generally
put, could be anything from a logo to a name to even shape of goods or combinations and as long
as it is capable of distinguishing the goods/services of the applicant in question, it is a trademark.
For Example:
A trademark is any
• Word (PEPSI, COCO COLA, and PEPSODENT),
• Name (RAYMOND VEIL, CHRISTIAN LOUBOUTIN),
• surname (TATA, BATA, and BAJAJ),
• Signature (LOUIS PHILIPPE),
• Letters (BMW, IBM, HDFC),
• Numeral (555, 501),
• Symbol (MERCEDES BENZ THREE POINTED STAR, ADDIDAS PYRAMID),
• Device (AMUL GIRL, PILLSBURY BOY),
• Color scheme (PIZZA HUT, DOMINOS),

109 

 
 

• packaging (CADBURY’S),
• Shape of goods (COCO COLA BOTTLE)
Or any combination thereof that identifies and distinguishes the source of the goods of one party
from those of others. When the mark is used in respect of services then the mark is also
commonly referred to as Service Mark. Ex: DHL, HILTON, INFOSYS are service marks.
However, since the distinction between goods and services are not always clear, the term
trademark is often used to include service marks as well as.
History of trademark-While some form of proprietary protection for marks in India dates
back several millennia, India’s statutory Trademarks Law dates back to 1860. Prior to 1940 there
was no official trademark Law in India. Numerous problems arouse on infringement, law of
passing off etc. and these were solved by application of section 54 of the specific relief act 1877
and the registration was obviously adjusted by obtaining a declaration as to the ownership of a
trademark under Indian Registration Act 1908.
To overcome the aforesaid difficulties the Indian Trademarks Act was passed in 1940.The
replacement to this act was the Trademark and Merchandise Act 1958.This Act was to provide
for registration and better protection of Trademarks and for prevention of the use of fraudulent
marks on merchandise.
The appellation of the Trademarks and Merchandise Act gave rise to the Trademark Act 1999;
this was done by the Government of India so that the Indian Trademark Law is in compliance
with the TRIPS obligation on the recommendation of the World Trade Organization. The object
of the 1999 Act is to confer the protection to the user of the trademark on his goods and prescribe
conditions on acquisition, and legal remedies for enforcement of trademark rights. The act of
1999 also gives police the right to arrest in case of infringement. There are some points of
changes that are present between the 1958 act and 1999 act, it can be said that the 1999 act is a
modification of the 1958 act, it has provided exhaustive definitions of terms frequently used,
enhanced punishment for offenders, increased the period of registration, registration of non-
traditional trademarks. The rules of this act are called as Trademark Rules 2002. Both the Act
and its set of rules came to effect on September 15th 2003. The trademark act 1999 and its
trademark rules 2002 presently govern Indian Trademark Laws in India.

110 

 
 

Global efforts for the protection of trademarks-


• Paris convention
• Madrid agreement and protocol
• Trademark law treaty
• Trips

TYPES OF TRADEMARK-
• PRODUCT TRADEMARKS-
Are those that are affixed to identify goods

•SERVICE TRADEMARKS-
Are used to identify the services of an entity, such as the trademark for a broadcasting service,
retails outlet, etc. They are used in advertising for services.

• CERTIFICATION TRADEMARKS-
Are those that are capable of distinguishing the goods or services in connection with which it is
used in the course of trade and which are certified by the proprietor with regard to their origin,
material, and the method of manufacture, the quality or other specific features

• COLLECTIVE TRADEMARKS-
Are registered in the name of groups, associations or other organizations for the use of members
of the group in their commercial activities to indicate their membership of the group.
Distinction between “Trade Mark” and “Property Mark”
There is a clear distinction in between Trade Mark and Property Mark is defined in sec 2(1) (zb)
of the Trade Mark Act1999. Whereas the Property Mark is explain in section 479 of the Indian
Penal Code, 1860. Initially the Trade Mark was defined in the Indian Penal Code, 1860 in its sec.
478, but it was repealed from the Penal Code, as a separate Act, The Trade and Merchandise
Act,1958 was been enacted.

111 

 
 

Q.3. Define trademark. What are the main characteristics of a good


trademark?
Ans- A trademark is any A trademark as per Section 2(zb) means “a mark capable of being
represented graphically and which is capable of distinguishing the goods or services of one
person from those of others and may include shape of goods, their packaging and combination of
colours:”
Trade Marks are like name of individuals. It comprises of a word, numeral, logo, picture,
signature, shape of an article, etc. or a combination of any or all of the above. They help the
customer identify the manufacturer or service provider of the goods / services. Therefore, they
are commonly referred to as brand names. examples:
1. Word – invented or dictionary
2. Letter (s)
3. Monograms
4. Name
5. Signature
6. Portrait of a person
7. Numeral
8. Alpha numeric combinations
9. Brand
10. Logo
11. Device
12. Scheme of colors applied to goods
13. Shape of goods
14. Heading
15. Label
16. Packaging
17. Pictorial representation of common objects like animals, birds, or other things
18. Ticket

112 

 
 

Global efforts for the protection of trademarks-


• Paris convention
• Madrid agreement and protocol
• Trademark law treaty
• Trips

CHARACTERISTICS OF A MARK-
1. Indication of origin– a trade mark should be indicative of the source of origin of the respective
goods i.e. be able to indicate the manufacturer of trader.
2. Distinctive – a trade mark should not refer to the kind, quality, nature, characteristics,
geographical origin of the respect goods and hence be distinctive in nature.
BENEFITS OF REGISTRATIONS-
Registration of trademarks is not mandatory under the Trade Marks Act, 1999. However, it is
advisable because:
1. Exclusive right to use the trade mark in relation to the goods or services in respect of which
the trade mark is registered.
2. The right to obtain relief in respect of infringement of the trade mark in the manner provided
in the Trade Mark Act, 1999.
3. Trade Mark Registration is a prima-facie proof of proprietorship in the Court of Law.

TERM OF REGISTRATION-
1. Once Registration is granted it is valid for ten years from the date of filing the application.
However, the rights in a mark accrue from the date of actual commercial use, or the date of
application whichever is earlier.
2. After the expiry of the first ten years the registration may be renewed periodically (10 yrs
each).

113 

 
 

SHORT QUESTION

Q.4. Describe the procedure for registration of trademark?


Ans-TRADEMARK REGISTRATION TENURE
It lasts 10 years and can be renewed for further period of 10 years. If you do not pay your
renewal fee by the next renewal date, your mark will expire. However, you are allowed an extra
six months from the renewal date in which to renew your registration, but you will have to pay a
fee for late renewal. Following this six-month period, there is a further six-month period, i.e. a
total of up to one year after the renewal date, in which you may apply to restore your mark.
Section 18 in the Trade Marks Act, 1999
Application for registration.—
(1) Any person claiming to be the proprietor of a trade mark used or proposed to be used by him,
who is desirous of registering it, shall apply in writing to the Registrar in the prescribed manner
for the registration of his trade mark.
(2) A single application may be made for registration of a trade mark for different classes of
goods and services and fee payable therefore shall be in respect of each such class of goods or
services.
(3) Every application under sub-section (1) shall be filed in the office of the Trade Marks
Registry within whose territorial limits the principal place of business in India of the applicant or
in the case of joint applicants the principal place of business in India of the applicant whose
name is first mentioned in the application as having a place of business in India, is situate:

114 

 
 

Provided that where the applicant or any of the joint applicants does not carry on business in
India, the application shall be filed in the office of the Trade Marks Registry within whose
territorial limits the place mentioned in the address for service in India as disclosed in the
application, is situate.
(4) Subject to the provisions of this Act, the Registrar may refuse the application or may accept it
absolutely or subject to such amendments, modifications, conditions or limitations, if any, as he
may think fit.

Process of registration-
Step 1: Trademark research
In the initial stage, you need to do the proper research before selecting a trademark for your
business. You need to follow the certain steps:
• Identify the mark to be registered as trademark with the registry
• Search the availability of the selected mark
• Identify the class of the trademark
Step 2: TM form filing
Once the uniqueness is achieved in mark while searching for the mark, you need to file an
application in form TM-A with the registrar for registration of a trademark. After filing the
application with the requisite fee and documents, the company can use the symbol ‘TM’ until the
trademark approved. A single trademark application can be filed for one or multi-class.
Step 3: Allotment of application number
After filing the trademark registration application with the Registrar, an application number will
be generated which helps you to track the application status online. After allotment of
application number, the applicant can use the ‘TM’ symbol with the selected mark.
Step 4: Examination of application
After trademark application filed at the trademark registrar office who has the jurisdiction over
the state, trademark examiner check the trademark application in accordance with the provisions
of Trademark Act. The registrar may accept or object the application on the relative or absolute
grounds.

115 

 
 

Step 5: Receipt of examination report


Once the trademark examiner reviews the application, an examination report issued by the
registry. Usually, report would be issued within 3 months to 1 year depending on the back log of
the registry.
Step 6: Reply to the examination report
Once you get the examination report, you need to file the reply with the registrar to escape the
objection raised by the trademark examiner. You need to file the reply within30 days counted
from the day of receiving the examination report. Non filing of the reply leads to abandon the
trademark application.
Step 7: Advertisement of trademarks journal
If no objection is raised by the registrar the or in case of objection, satisfied with the reply, the
trademark registrar will publish the mark in trademark journal. The filed trademark application is
advertised in trademark journal to invite the public to oppose for the mark.
Step 8: Opposition (if any)
After publishing the trademark the third party have an opportunity to raise the objection within
90 days from the date of publication. If the third party opposed the trademark application then
both the parties have the chance to appear at the hearing.
Step 9: Certificate of registration
When objections and oppositions (if any) are satisfied which are raised by the examiner or third
party, then trademark registration certificate issued to the applicant. Registration of the
trademark granting the exclusive use to the owner of the registered trademark. For registered
trademark symbol ‘R’ has to be used.

Q.5. Discuss passing off?


Ans-Passing off is a common law tort, which can be used to enforce unregistered trademark
rights. The law of passing off prevents one person from misrepresenting his goods or services as
that of another.
The concept of passing off has undergone changes in the course of time. At first it was restricted
to the representation of one person's goods as those of another. Later it was extended to business

116 

 
 

and services. Subsequently it was further extended to professions and non-trading activities.
Today it is applied to many forms of unfair trading and unfair competition where the activities of
one person cause damage or injury to the goodwill associated with the activities of another
person or group of persons.
In British Diabetic Association V Diabetic Society 1, both the parties were charitable societies.
Their names were deceptively similar. The words 'Association' and 'Society' were too close since
they were similar in derivation and meaning and were not wholly dissimilar in form. Permanent
injunction granted.

Elements of Passing Off


The three fundamental elements of passing off are Reputation, Misrepresentation and Damage to
goodwill. These three elements are also known as the CLASSICAL TRINITY, as restated by the
House of Lords in the case of Reckitt & Colman Ltd V Borden Inc . It was stated in this case that
in a suit for passing off the plaintiff must establish firstly, goodwill or reputation attached to his
goods or services. Secondly he must prove a misrepresentation by the defendant to the public i.e.
leading or likely to lead the public to believe that the goods and services offered by him are that
of the plaintiff's. Lastly he must demonstrate that he has suffered a loss due to the belief that the
defendant's goods and services are those of the plaintiff's.

Modern Elements of Passing off - In the case Erven Warnink Vs. Townend 3 , Lord Diplock
gave the essential modern characteristics of a passing off action. They are as follows: -
1. Misrepresentation
2. Made by a person in the course of trade
3. To prospective customers of his or ultimate consumers of goods or services supplied by him.
4. Which is calculated to injure the business or goodwill of another trader.
5. Which causes actual damage to a business or goodwill of the trader by whom the action is
brought.

117 

 
 

The above concept of passing off can be explained with the help of few case laws: Honda Motors
Co. Ltd V Charanjit Singh & Others
Held: It was held that the use of the mark "Honda" by the defendants couldn't be said to be an
honest adoption. Its usage by the defendant is likely to cause confusion in the minds of the
public. The application of the plaintiff was allowed.

Q. 6. Why passing off is necessary ?


The Trademark is providing protection to registered goods and services, but the passing off
action is providing a protection to unregistered goods and services. The most important point is
that the remedy is same in both the cases but the Trademark is available to only the registered
goods and services and passing off is available to unregistered goods and services. To more
knowledge of this context we can summaries the case of Durga Dutt vs. Navaratna
Pharmaceutical[2]; in this case the Supreme Court is set out the distinction between infringement
and passing off. The action for infringement is a statutory remedy conferred on the registered
owner of a registered Trade mark and has an exclusive right to the use of the trade mark in
relation to those goods. And the passing off is available to the unregistered goods and services.
The second most important point is that the use by the defendant of the trade mark of the plaintiff
is not essential in an action for passing off, but in the case of an action for infringement this will
not applicable.
The third important distinction between these two is that if the essential features of the trade
mark of the plaintiff have been adopted by the defendant, the fact that the get up, packing and
other writing or marks on the goods or on the packets in which he offers his goods for sale
marked differences or indicate clearly a trade origin different from that of the registered owner of
the mark would be immaterial; but in case of passing off the defendant may escape liability if he
can show that the added matter is sufficiently to distinguish his goods from those of the plaintiff.
In the cases of infringement the burden is always lies to the plaintiff. In the case of S.M.
Dyechem Ltd. v. Cadbury (India) Ltd.[4] In this case an infringement action is fail where
plaintiff cannot prove registration or that its registration extends to the goods or to all the goods
in question or because the registration is invalid and yet the plaintiff may show that by imitating

118 

 
 

the mark otherwise, the defendant has done what is calculated to pass off his goods as those
plaintiff.
When the passing off arise
The passing off action is arise when there is misrepresentation, when it is harm the existence
plaintiff’s goodwill, when it is made by a trader in the course of trade, which is injure the
business of another trader and which cause actual damage to the business or goodwill of the
trader by the whom action is brought.
But these requirements were reduced to three in Reckitt & Colman Products Ltd. V. Borden Inc.
now there are three essential requirements for the passing off action:
The Claimant’s Goodwill: Although damage is the gist of an action for passing off, but the
plaintiff must show that there is a reasonable reason of his being injured by the defendant’s
action, even if the conduct of the defendant might be calculated to deceive the public. A private
individual cannot institute a suit for passing off even if the defendant practices deception upon
the public, unless it is proved that the defendant’s action is likely to cause damage to the
individual.
v Misrepresentation: Misrepresentation in the simplest form of passing off. If A says falsely
these goods I am selling are B’s goods. It is a clear case of passing off. In simple way we can say
that misrepresentation should lead. Or be likely to lead confusion on the part of consumers. In
case ofKhemraj v. Garg, in this case the defendants had copied the get up, layout, design and
colour scheme, etc. and the name “manavpanchang,mani ram panchang” and “shri vallabh Mani
Ram panchang” of the plaintiff’s panchang.The court held that it is similar to the plaintiff’s
product and Interim injunction was granted.

The case of Akash Arora vs. Yahoo In, in this case the court held that the yahooindia is creating
a confusion in the mind of the people. And the defendant yahooindia is same as the plaintiff’s
yahoo. But as a student of law I am not go with the case decision because my views regarding to
the case is that his site may be better than his competitors. And second important thing is that
those who access the Internet they are capable to distinguish which site is yahoo. in and which

119 

 
 

one is yahooindia.So the question of confusion is not create when the people are able to
distinguish between the sites then there will be no question of passing off arise.
The second case In Reckitt & Colman of India Ltd. vs. M.P. Ramachandran & Anr, Humble
Calcutta High Court (Barin Ghosh, J.) laid down five principles for granting an injunction in case
of comparative advertising:
i. A tradesman is entitled to declare his goods to be best in the world even though the declaration
is untrue;
ii. He can also say that his goods are better than his competitors, even though such statement is
untrue;
iii. For the purpose of saying that his goods are the best in the world or his goods are better than
his competitors he can even compare the advantages of his goods over the goods of others;
iv. He however, cannot, while saying that his goods are better than his competitors, say that his
competitor’s goods are bad. If he says so, he really slanders the goods of his competitors and
their goods, which is not permissible.
Reverse passing off
It will be recalled that orthodox passing off entails the defendant representing that his product is
the plaintiff's product. In many cases, reverse passing off can be explained under the ordinary
rules: for example where a defendant may represent that he or she made goods which were in
fact made by the plaintiff so as to pass off his own business as a branch of the plaintiff's.

Conclusion
In the conclusion the researcher concludes that the passing off action is applied in unregistered
goods and services, and in infringement of suit and passing off in both the cases the remedy will
be same. Then the passing off is arise in three cases first when it is injured the claimants good
will, secondly in misrepresentation and thirdly in damages, where the position is same like in
infringement suit. And lastly when the concept of passing off is reading with domain name and
technological changes then the concept of passing off is in different dimension.

120 

 
 

VERY SHORT QUESTION


Q.7. write a short note on honest concurrent user?
Ans-Honest And Concurrent Use of Trademark-“Honest concurrent use”, as laid down under
section 12 of the Trade Marks Act, 1999, is one of those grounds prescribed under which
identical or similar marks by more than one proprietor and in relation to same or similar
goods/services may be permitted registration by the Registrar, based upon his subjective
discretion. The section also makes mention of “other special circumstances” in addition to the
honest concurrent defense.
A plain, cursory reading of the law evidently hints that the Registrar is in no way obliged to
register the disputed mark, and that much relies on how the applicant/defendant goes about
proving honest and concurrent use of the contesting mark.
A Brief Background of the law of ‘Honest Concurrent Use’ and its Evolution
Two of the earliest cases (of the 1860s) in this context are known to have led to the emergence of
the above rule, those being, Dent vs. Turpin andSouthorn vs. Reynolds.Subsequently, in the
English case John Fitton & Co. [(1949) 66 RPC 110,112], five factors were laid down as
conditions necessary to be fulfilled in order to lend credence to the honest concurrent user
defense:
In most jurisdictions, Trade Mark law provides for a Trade Mark of an honest and concurrent
user to co exist with another similar mark. The defence of honest and concurrant user was
conceived mainly through two cases. Firstly, the case of Dent v Turpin i determined that two
users of a mark (which had derived from a common predecessor) had a separate right to obtainan
injunction against a third person using the mark. Secondly, in Southorn v Reynolds ii , the Dent
case was relied on to come to a conclusion on very similar facts. Though, it should be noted that
neither of these two cases was related to dispute between concurrent users. The courts, in these
two cases were not protecting the exclusive property rights but restraining a person from
misrepresenting his goods as those of another.
In John Fitton & Co, following five factors were laid down in order to have success in honest
and concurrent user defense:
1. The extent of use in time and quantity and the area of the trade

121 

 
 

2. The degree of confusion likely to ensue from the resemblance of the marks which is to a large
extent indicative of the measure of public inconvenience.
3. The honesty of the concurrent use
4. Whether any instances of confusion have infact been proved

Q.9. Explain property mark?


Ans- section 479 of IPC-A mark used for denoting that movable property belongs to a particular
person is called a property mark. It means that marking any movable property or goods, or any
case, package or receptacle containing goods; or using any case, package or receptacle, with any
mark thereon.
Example: The mark used by the Indian Railway on their goods may be termed as a Property
Mark for the purpose of easy identification of the owner.

Q. 10. Explain the duration of trademark?


Ans-Section 25 in The Trade Marks Act, 1999- Duration, renewal, removal and restoration of
registration.—
(1) The registration of a trade mark, after the commencement of this Act, shall be for a period of
ten years, but may be renewed from time to time in accordance with the provisions of this
section.
(2) The Registrar shall, on application made by the registered proprietor of a trade mark in the
prescribed manner and within the prescribed period and subject to payment of the prescribed fee,
renew the registration of the trade mark for a period of ten years from the date of expiration of
the original registration or of the last renewal of registration.
(3) At the prescribed time before the expiration of the last registration of a trade mark the
Registrar shall send notice in the prescribed manner to the registered proprietor of the date of
expiration and the conditions as to payment of fees and otherwise upon which a renewal of

122 

 
 

registration may be obtained, and, if at the expiration of the time prescribed in that behalf those
conditions have not been duly complied with the Registrar may remove the trade mark from the
register: Provided that the Registrar shall not remove the trade mark from the register if an
application is made in the prescribed form and the prescribed fee and surcharge is paid within six
months from the expiration of the last registration of the trade mark and shall renew the
registration of the trade mark for a period of ten years under sub-section (2).
(4) Where a trade mark has been removed from the register for non-payment of the prescribed
fee, the Registrar shall, after six months and within one year from the expiration of the last
registration of the trade mark, on receipt of an application in the prescribed form and on payment
of the prescribed fee, if satisfied that it is just so to do, restore the trade mark to the register and
renew the registration of the trade mark either generally or subject to such conditions or
limitations as he thinks fit to impose, for a period of ten years from the expiration of the last
registration.

Q.11. Distinction between trademark and property mark?

1.)A property mark attached to the movable property of a person remains even if part of such
property goes out of his hands and ceases to be his.
A trade mark means a mark used in relation to goods for the purpose of indicating or so as to
indicate a connection in the course of trade between the goods and some person having the right
as proprietor to use that mark.

2.) A property mark, as defined by Section 479 of the Penal Code means a mark used for
denoting that a movable property belongs to a particular person.

123 

 
 

A trade mark, as defined by Section 2(1)(j) of the Trade and Merchandise Marks Act, 1958,
includes a device, brand, heading, label, ticket, name, signature, word, letter or numerical or any
combination thereof.

3.) A trade mark concerns the goods themselves


A property mark concerns the proprietor.

124 

You might also like